[obm-l] Const. de triângulo

2024-01-13 Por tôpico Luís Lopes
Saudações, oi Anderson,

Soluções usando fórmulas servem para mostrar que o triângulo é construtível e 
qual é sua forma e tamanho. Já ajuda naquela parte - suponha o problema 
resolvido. Mas a construção procurada deverá ser feita usando as propriedades 
da figura.

Posso mandar no privado para quem se interessar as construções com as figuras 
que um correspondente me enviou. Esse que tem h_c/b como dado é bem 
interessante.

Agora o problema  pode ser resolvido de 3 ou mais maneiras. Com 
medianas é sempre bom pensar em simetrias e paralelogramos.

Luís


-- 
Esta mensagem foi verificada pelo sistema de antiv�rus e
 acredita-se estar livre de perigo.


=
Instru��es para entrar na lista, sair da lista e usar a lista em
http://www.mat.puc-rio.br/~obmlistas/obm-l.html
=


[obm-l] Const. de triangulo

2024-01-11 Por tôpico Luís Lopes
Mando outra mensagem pois reply não funciona. Pensei que estava claro. Notação 
padrão de triângulo. Construir os triângulos com R com os dados fornecidos.
h_a altura; m_a mediana; 
b+c soma dos lados AC e AB (vértices do triângulo);
h_c:b razão h_c/b



-- 
Esta mensagem foi verificada pelo sistema de antiv�rus e
 acredita-se estar livre de perigo.


=
Instru��es para entrar na lista, sair da lista e usar a lista em
http://www.mat.puc-rio.br/~obmlistas/obm-l.html
=


[obm-l] Const. de triangulo

2024-01-11 Por tôpico Luís Lopes
Vou mandar um texto bem carequinha.

h_a,m_a,h_c:b
b+c,h_a,h_b:h_c

Luís


-- 
Esta mensagem foi verificada pelo sistema de antiv�rus e
 acredita-se estar livre de perigo.


=
Instru��es para entrar na lista, sair da lista e usar a lista em
http://www.mat.puc-rio.br/~obmlistas/obm-l.html
=


[obm-l] Teste

2024-01-11 Por tôpico Luís Lopes
Mensagens não chegam.
-- 
Esta mensagem foi verificada pelo sistema de antiv�rus e
 acredita-se estar livre de perigo.


=
Instru��es para entrar na lista, sair da lista e usar a lista em
http://www.mat.puc-rio.br/~obmlistas/obm-l.html
=


[obm-l] f(f(x))=e^(-x)

2023-09-24 Por tôpico Luís Lopes
Minhas mensagens não chegam quando faço reply. Tenho sempre que começar uma 
nova. Segue a que mandei ontem, agora incluindo o Gugu (parece que é assim que 
ele gosta de ser chamado).

%%

Saudações,

Obrigado aos que responderam. É por aí, Ralph. Seu argumento é quase uma cópia 
do que veio no e-mail. Coloco aqui sem editá-lo. Vou encaminhar sua resposta ao 
correspondente (não o conheço). Obrigado novamente. 
Luís

===
Navegando pela Internet Youtube encontrei um desafio atinente
à composição de FRVR .A  F função seguinte:  (FoF)(X) = e^(-X)
Após mostrar que a função exponencial é decrescente  e^(-X)
Se X(FoF)(Y)   é verdadeiro!  FD.
Se Xhttp://www.mat.puc-rio.br/~obmlistas/obm-l.html
=


[obm-l] f(f(x))=e^(-x)

2023-09-23 Por tôpico Luís Lopes
Saudações,

Existe tal f? Se sim, qual seria?

Recebi um e-mail com esta pergunta, sem maiores detalhes. Pelo e-mail, tal f 
não existe. Problema encontrado pelo remetente no YouTube.

Luís


-- 
Esta mensagem foi verificada pelo sistema de antiv�rus e
 acredita-se estar livre de perigo.


=
Instru��es para entrar na lista, sair da lista e usar a lista em
http://www.mat.puc-rio.br/~obmlistas/obm-l.html
=


[obm-l] Comunicação

2023-09-17 Por tôpico Luís Lopes
Data: 25/08/2023
De: Priscila Santana 
Para: obm-l@mat.puc-rio.br
Assunto: [obm-l] Comunicação
Olá!

Existe algum grupo de discussão de questões olímpicas no WhatsApp?

Atte.


Priscila S. da Paz

Sauda,c~oes,

O Renato Madeira administra um. Ele pede somente que quem pedir
para se inscrever que se identifique cono membro desta lista.

Renato Madeira
WhatsApp: 55 21 99889 1516

Luís Lopes


-- 
Esta mensagem foi verificada pelo sistema de antivírus e
 acredita-se estar livre de perigo.



[obm-l] construir triângulos dados

2022-10-09 Por tôpico Luís Lopes
Sauda,c~oes,

O problema terá duas soluções (h_a=h=altura e d_a=d=bissetriz interna, k=d/h) se
0https://www.wolframalpha.com/input?i=0%3CB%3C%3Dpi%2F3%3B+1%3Ck%3C%5Csec%28B%2F2%29%3B+tan%5E2B%3D4%28k%5E2-1%29%2F%28k%5E4-4k%5E2%2B4%29

https://www.wolframalpha.com/input?i=pi%2F3%3CB%3Cpi%2F2%3B+1%3Ck%3C%5Ccsc%28B%29%3B+tan%5E2B%3D4%28k%5E2-1%29%2F%28k%5E4-4k%5E2%2B4%29

Obrigado.
Luís



-- 
Esta mensagem foi verificada pelo sistema de antivírus e
 acredita-se estar livre de perigo.



[obm-l] Eliminar parâmetro t

2020-11-16 Por tôpico Luís Lopes
Sauda,c~oes,

Num problema de encontrar o lugar geométrico do vértice
A de um triângulo, encontrei como valores das coordenadas
x_A e y_A as seguintes expressões:

A(x_A,y_A) com  x_A = N/D,  y_A=P/D, onde N=m(v^2+t^2);
D=t(1+m^2); P=m^2 v^2 - t^2.

Fora t, que tem que ser eliminado, todos os outros parâmetros
são fixos e conhecidos.

Não consegui fazer, obtendo sempre uma identidade 0=0.

Um amigo que já me ajudou nessas questões mandou a resposta,
obtida por computador. Para facilitar meus cálculos, tinha feito a=2v.
Daí o  na fórmula enviada:

-a^2 m + 4 m x^2 - 4 x y + 4 m^2 x y - 4 m y^2 = 0.

O lugar geométrico é uma hipérbole equilátera. O locus está correto.

Como fazer isso ? Outras eliminações mais difíceis que ele me enviou
eu nem tentaria fazer à mão. Mas essa não parecia difícil.
Como fazer ? Qual a técnica ? Deve haver uma para o computador e
casos complicados.

Luís



[obm-l] polinômio irredutível

2020-08-16 Por tôpico Luís Lopes
Sauda,c~oes,

Como provar que um polinômio f(x) tendo como coeficientes números inteiros
é irredutível se e somente se f(x+a) é irredutível para algum  inteiro ?

Luís




-- 
Esta mensagem foi verificada pelo sistema de antivírus e
 acredita-se estar livre de perigo.



RE: [obm-l] teste

2020-08-08 Por tôpico Luís Lopes
Sauda,c~oes,

Oi Carlos Victor,

Obrigado por responder. Acho que recebi sua mensagem pois vc a enviou
tanto para a lista quanto para mim diretamente. E do mesmo jeito, acho que
você vai receber minha resposta somente porque estou escrevendo
diretamente pra você, com cópia pros dois endereços da obm.puc.

Isso já acontece comigo há muito, muito tempo. No começo quando fazia reply,
agora quando mando uma mensagem nova. Às vezes mando as mensagens por
um outro email, e essas eu vejo chegarem no email que mandei. Mas não chegam
nesse daqui. E muito certamente não chegam na lista também.

Se por acaso alguém da lista receber esta mensagem e puder entrar em contato
com o administrador, ficaria muito grato.

Obrigado.

Luís





De: Carlos Victor 
Enviado: sábado, 8 de agosto de 2020 17:01
Para: obm-l@mat.puc-rio.br 
Cc: owner-ob...@mat.puc-rio.br ; Luís Lopes 

Assunto: Re: [obm-l] teste


Há muito tempo que os meus emails enviados também estão assim e não sei o 
motivo.

Carlos Victor

PS : este email não sei se chegará aos companheiros da lista

Em 08/08/2020 17:39, Luís Lopes escreveu:

Recebo as mensagens normalmente. Mas não tenho confirmação de
chegada ao grupo das que envio. E não aparecem nos arquivos também.

Mandei uma há umas 5 horas intitulada polinômio minimal.
Chegou? Alguém recebeu ?

Luís


--
Esta mensagem foi verificada pelo sistema de antivírus e
acredita-se estar livre de perigo.


-- 
Esta mensagem foi verificada pelo sistema de antivírus e
 acredita-se estar livre de perigo.



[obm-l] teste

2020-08-08 Por tôpico Luís Lopes
Recebo as mensagens normalmente. Mas não tenho confirmação de
chegada ao grupo das que envio. E não aparecem nos arquivos também.

Mandei uma há umas 5 horas intitulada polinômio minimal.
Chegou? Alguém recebeu ?

Luís


-- 
Esta mensagem foi verificada pelo sistema de antivírus e
 acredita-se estar livre de perigo.



[obm-l] construção geométrica

2020-06-10 Por tôpico Luís Lopes
Sauda,c~oes,

Recebi o seguinte problema:

Construir P no circuncírculo de um triângulo ABC dado
tal que PA+PB=PC.

Alguém saberia fazer ?

Obrigado.

Abs,
Luís



-- 
Esta mensagem foi verificada pelo sistema de antivírus e
 acredita-se estar livre de perigo.



[obm-l] construir triângulo dados , ,

2020-05-11 Por tôpico Luís Lopes
Sauda,c~oes,

d_a : bissetriz interna do vértice A ; ADa = d
e_a : bissetriz externa do vértice A; AEa = ea
m_b : mediana BMb = m
b:c=b/c=k

Os problemas  e  podem ser construídos
com régua e compasso usando o teorema das bissetrizes e as
propriedades da divisão harmônica. Como dica, ver o problema 7
na página 12 do livro Geometria II do Wagner/Morgado/M. Jorge de 1974.

Mas só consegui construir o triângulo ABC no caso 
usando álgebra.

Resolvendo as equações, obtive um valor para  que é construtível
com régua e compasso:

c=sqrt(4m2 + 2d2(k+1)2/k)/(k+2) = sqrt(4m2 + 2d2(k+1)2k-1)/(k+2)

Embora aceitável, estas soluções são muito rotineiras e não acrescentam
nada aos problemas, não usam nenhuma propriedade das figuras. Ou seja,
deixam a desejar do ponto de vista das construções geométricas propriamente.

Alguém poderia propor uma construção elegante, usando as propriedades da
figura e os teoremas da geometria ?

Como exemplo, considere o problema . Resolva-o das duas
maneiras e você verá o que quero dizer.

Dica: ver os resultados das seções 1.9, 2.8 e a figura da página 271 da
referência acima. Como mostrado no livro Geometria I dos mesmos
autores de 1990, página 74--75, MR=b-c.

Abraço,
Luís


-- 
Esta mensagem foi verificada pelo sistema de antivírus e
 acredita-se estar livre de perigo.



[obm-l] RE: Logaritmos e Sequência

2020-03-06 Por tôpico Luís Lopes
Sauda,c~oes, oi Maikel,

Escrevi três páginas sobre isso no livro
Manual das Funções Exponenciais e Logarítmicas.

Luís



De: owner-ob...@mat.puc-rio.br  em nome de Maikel 
Andril Marcelino 
Enviado: sexta-feira, 6 de março de 2020 01:26
Para: obm-l@mat.puc-rio.br 
Assunto: [obm-l] Logaritmos e Sequência


Boa noite, pessoal! Estou fazendo um trabalho. Meu orientador afirmou que havia 
uma maneira de introduzir o conceito de logaritmo com progressões A. e G.. Na 
minha graduação eu elaborei uma aula, que abordava progressões, porém era sobre 
propriedades de logaritmos. Algum ser humano tem ideia de como abordar o 
conceito de logaritmos por meio de sequências?


Atenciosamente,

Maikel Andril Marcelino
(84) 9-9149-8991 (Contato)
(84) 8851-3451 (WhatsApp)

--
Esta mensagem foi verificada pelo sistema de antivírus e
acredita-se estar livre de perigo.

-- 
Esta mensagem foi verificada pelo sistema de antivírus e
 acredita-se estar livre de perigo.



[obm-l]

2020-02-14 Por tôpico Luís Lopes
Minhas mensagens não estão chegando. Tento mais uma vez.

Sauda,c~oes,

Construir o triângulo (sinteticamente, sem (muita) álgebra)
com os dados acima. k é um número real (construtível) conhecido.

 Não sei se pode servir como aquecimento mas o problema
 me parece mais fácil.

Fonte: Il Problema Geometrico Dal compasso al Cabri.
Italo D'Ignazio e Ercole Supra.

O segundo aparece no Petersen também.

Os problemas  e  são casos particulares com k=1.

Abraços,
Luís


-- 
Esta mensagem foi verificada pelo sistema de antivírus e
 acredita-se estar livre de perigo.



[obm-l] construção geométrica

2020-01-19 Por tôpico Luís Lopes
Sauda,c~oes,

Envio o enunciado do problema tal como está no livro.

Construct a triangle, which shall have its vertex in a
given line, having a given base and a given difference
of the angles at the base.

Fonte: Julius Petersen, Methods and Theories for the
solution of Problems of Geometrical Constructions, 1927.
Problema 327, p.59.

Abraços,
Luís


-- 
Esta mensagem foi verificada pelo sistema de antivírus e
 acredita-se estar livre de perigo.



[obm-l] soma com cevianas que passam pelo circuncentro

2019-12-19 Por tôpico Luís Lopes
Sauda,c~oes,

Encontrei um link com a prova:

https://www.cut-the-knot.org/m/Geometry/CeviansThroughCircumcenter.shtml

Esse site é muito bom.

Eu conhecia a prova 3 mas não sabia que o triângulo tinha que ser acutângulo.
Para triângulo retângulo vale também, por verificação direta.

Aí comecei a rever a prova para triângulos obtusângulos e vi que
havia um problema com (B-C)=90º. Acho que para triângulos obtusângulos
a igualdade pode valer mas tem que ver para quais casos ela
não serve. Talvez (B-C) > 90º como (115º,15º,50º) e (B-C) < 90º
como (105º,45º,30º) satisfazem mas (B-C) = 90º como (120º,30º,30º)
não satisfaz. Isso precisaria de outra investigação.

Abraços,
Luís





-- 
Esta mensagem foi verificada pelo sistema de antivírus e
 acredita-se estar livre de perigo.



[obm-l] soma com cevianas que passam pelo circuncentro

2019-12-18 Por tôpico Luís Lopes
Sauda,c~oes,

Sejam AO_a, BO_B e CO_c as cevianas que passam pelo circuncentro.
O_a na reta do lado  etc.

Como provar que 1/AO_a + 1/BO_b + 1/CO_c = 2/R ?

Luís


-- 
Esta mensagem foi verificada pelo sistema de antivírus e
 acredita-se estar livre de perigo.



[obm-l] polinômio redutível ?

2019-11-05 Por tôpico Luís Lopes
Sauda,c~oes,

Considere o polinômio

p(x)[h,m,s] = 9x^4 + 12s x^3 + 2(8h^2 - 20m^2 - s^2)x^2 + 4s(4m^2 - s^2)x + 
(4m^2 - s^2)^2 .

Fiz alguns testes para ver se p(x) pode ter suas raízes construtíveis. Esse 
polinômio aparece
na construção do triângulo dados .

1) h=4sqrt(3); m=sqrt(21); s=13

p(x) = 9x^4 + 156x^3 - 410x^2 - 4420x + 7225 = (x-5)(3x+17)(3x^2 + 50x - 85)

2 positivas e 2 negativas

2) h=4sqrt(3); m=2sqrt(3); s=13

p(x) = 9x^4 + 156x^3 - 50x^2 - 6292x + 14641 = (3x^2 + 26x - 121)^2

1 positiva e 1 negativa raízes duplas

3) h=4sqrt(3); m=(3/2)sqrt(3); s=13

p(x) = 9x^4 + 156x^3 + 160x^2 - 7384x + 20164 = ? (Ax^2 + Bx + C)(Dx^2 + Ex + F)

irredutível  4 raízes complexas

x_1 = -13/3 - 2 i sqrt(7/3) - 1/2 sqrt(2044/9 + 208/3 i sqrt(7/3))

x_2 = -13/3 + 2 i sqrt(7/3) - 1/2 sqrt(2044/9 - 208/3 i sqrt(7/3))

x_3 = -13/3 - 2 i sqrt(7/3) + 1/2 sqrt(2044/9 + 208/3 i sqrt(7/3))

x_4 = -13/3 + 2 i sqrt(7/3) + 1/2 sqrt(2044/9 - 208/3 i sqrt(7/3))

4) h=12; m=7; s=13

p(x) = 9x^4 + 156x^3 + 6x^2 + 1404x + 729 = 3 (3x^4 + 52x^3 + 2x^2 + 468x + 
243) =
(3 x^2 - (8 sqrt(13) - 26) x + 27) (3 x^2 + (26 + 8 sqrt(13)) x + 27)

irredutível  2 raízes complexas 2 negativas

5) h=13; m=7; s=13

p(x) = 9x^4 + 156x^3 + 406x^2 + 1404x + 729 = 1/9 (-3 x - 2 i sqrt(39 sqrt(3) - 
49) + 6 sqrt(3) - 13) (-3 x + 2 i sqrt(39 sqrt(3) - 49) + 6 sqrt(3) - 13)   (3 
x - 2 sqrt(49 + 39 sqrt(3)) + 6 sqrt(3) + 13) (3 x + 2 sqrt(49 + 39 sqrt(3)) + 
6 sqrt(3) + 13)

= 1/9(9 x^2 - 36 sqrt(3) x + 78 x + 81)(9 x^2 + 36 sqrt(3) x + 78 x + 81)

irredutível  2 raízes complexas 2 negativas

Só testei para h,m,s > 0 mas se não errei nessas contas parece que podemos 
fatorar p(x)[h,m,s]  como

9x^4 + 12s x^3 + 2(8h^2 - 20m^2 - s^2)x^2 + 4s(4m^2 - s^2)x + (4m^2 - s^2)^2  = 
(Ax^2 + Bx + C)(Dx^2 + Ex + F) com os coeficientes A,B,... F construtíveis.

Daria para calcular os coeficientes dos dois polinômios do segundo grau em 
função de h,m,s ?

Luís




-- 
Esta mensagem foi verificada pelo sistema de antivírus e
 acredita-se estar livre de perigo.



[obm-l] Retas envolvendo uma parábola

2019-10-11 Por tôpico Luís Lopes
Sauda,c~oes,

Numa troca de mensagens sobre um procedimento de resolução do problema
"construir um triângulo ABC dados " apareceu a seguinte afirmação:

When two points move along two intersecting
straight lines at constant speeds (not necessarily equal)
line connecting them envelopes a parabola.

Alguém poderia me dar a prova ou referências sobre isso ?

Obrigado.

Luís



-- 
Esta mensagem foi verificada pelo sistema de antivírus e
 acredita-se estar livre de perigo.



[obm-l] problema de contagem

2018-11-04 Por tôpico Luís Lopes
Sauda,c~oes, oi Anderson,

Não precisa pensar nisso, é um problema de contagem.

Quero enunciar com palavras todos os problemas possíveis.

Exemplos:

(A,B,C) - dados os três ângulos
(A,B,a) - dados dois ângulos e o lado oposto a um deles
Assim (A,B,b) não conta.
(A,B,c) - dados dois ângulos e o lado oposto ao terceiro
(A,a,b) - dados um ângulo, o lado oposto ao ângulo e outro lado
(A,b,c) - dados um ângulo e os lados que formam este ângulo
(h_a,d_a,d_b) - altura e bissetriz interna partindo do mesmo vértice
e uma outra bissetriz interna
(d_c,d_b,h_b) - não conta, idêntico ao acima

Abraços,
Luís


-- 
Esta mensagem foi verificada pelo sistema de antivírus e
 acredita-se estar livre de perigo.



[obm-l] problema de contagem

2018-10-30 Por tôpico Luís Lopes
Sauda,c~oes,


Construir um triângulo dados três quaisquer dos seguintes elementos:


A,B,C - ângulos

a,b,c - lados

h_a,h_b,h_c - alturas

m_a,m_b,m_c - medianas

d_a,d_b,d_c - bissetrizes internas

e_a,e_b,e_c - bissetrizes externas

R - circunraio
r - inraio

r_a,r_b,r_c - exraios


Quantos problemas diferentes podem ser criados ?


Problemas do tipo (A,B,C) e (A,a,R) são considerados. Assim como

(A,B,a) e (A,B,c). Mas (A,B,a) e (A,B,b) contam como um só.


Abs,

Luís








-- 
Esta mensagem foi verificada pelo sistema de antivírus e
 acredita-se estar livre de perigo.



[obm-l] Const. triângulo dados "B=2C,a,b-c" e "B=2C,b,c"

2018-09-16 Por tôpico Luís Lopes
Sauda,c~oes,

Oi Claudio,

Obrigado.

< Com centro em A trace um círculo de raio c,
< intersectando o diâmetro do semi-círculo original em B.
De modo geral, o círculo (A,c) terá duas interseções
B1 e B2 com o diâmetro. E somente uma delas serve.
Trace a bissetriz (d1) de um deles (B1) e seja D1 a
interseção de d1 com AC. Se o triângulo B1CD1 é
isósceles, então B1 é a solução. Do contrário, B2.

Abs,
Luís


-- 
Esta mensagem foi verificada pelo sistema de antivírus e
 acredita-se estar livre de perigo.



[obm-l] Re: Const. triângulo dados "B=2C,a,b-c" e "B=2C,b,c"

2018-09-15 Por tôpico Luís Lopes
Sauda,c~oes,


Já que ninguém respondeu, a construção de ambos os problemas

é imediata sabendo-se que b^2=ac + c^2 .


A construção apresentada no FGM para o 1º problema

não precisou usar essa informação.


Abs,

Luís






De: owner-ob...@mat.puc-rio.br  em nome de Luís 
Lopes 
Enviado: sexta-feira, 14 de setembro de 2018 13:25:39
Para: obm-l@mat.puc-rio.br
Assunto: [obm-l] Const. triângulo dados "B=2C,a,b-c" e "B=2C,b,c"


Sauda,c~oes,


No livro do FGM de Trigonometria o 1º problema

tem uma construção somente por geometria.


Já o 2º encontrei num livro dos anos 50 que comprei

num sebo. O autor é Plácido Loriggio. Não tem a

construção nem sugestão. Procuro uma solução

puramente geométrica.


Abs,

Luís




--
Esta mensagem foi verificada pelo sistema de antivírus e
acredita-se estar livre de perigo.

-- 
Esta mensagem foi verificada pelo sistema de antivírus e
 acredita-se estar livre de perigo.



[obm-l] Const. triângulo dados "B=2C,a,b-c" e "B=2C,b,c"

2018-09-14 Por tôpico Luís Lopes
Sauda,c~oes,


No livro do FGM de Trigonometria o 1º problema

tem uma construção somente por geometria.


Já o 2º encontrei num livro dos anos 50 que comprei

num sebo. O autor é Plácido Loriggio. Não tem a

construção nem sugestão. Procuro uma solução

puramente geométrica.


Abs,

Luís




-- 
Esta mensagem foi verificada pelo sistema de antivírus e
 acredita-se estar livre de perigo.



[obm-l] Soma de Produtos de Termos em PA

2018-08-30 Por tôpico Luís Lopes
Sauda,c~oes, oi Claudio,

Seja S_{k-1} = (n-1)/(a1*an) = \frac{n-1}{a_1a_n}.

Para provar a recíproca escrevi

S_k = S_{k-1} + \frac{1}{a_n a_{n+1}} = \frac{n}{a_1a_{n+1}}

e cheguei a

n(a_{n+1} - a_n)=a_{n+1} - a_1 (*).

Fazendo a) n=2 e b) n=3 em (*) tem-se

a) a_3 + a_1 = 2a_2

b) a_4 + a_2 = 2a_3

Mas não consegui provar que a_{k+1} + a_{k-1} = 2a_k .

Usando (*) ou de outra maneira, como provar a recíproca ?

[]s
Luís



-- 
Esta mensagem foi verificada pelo sistema de antivírus e
 acredita-se estar livre de perigo.



[obm-l] caminho mínimo (suite 1)

2018-05-26 Por tôpico Luís Lopes
Realmente as mensagens não aparecem (?) quando
respondo na continuação da última recebida. Então
abro (recomeço) uma nova.

Sauda,c~oes, oi Pedro José,

A construção apresentada diz que OP tem que ser a
bissetriz do ângulo APB. Por quê ? Chega-se a isso
fazendo contas e interpretando o resultado geometricamente ?

Daí se U é o conjugado harmônico de O em relação a AB
e Gamma é o círculo de diâmetro OU, então o(s) ponto(s)
procurado(s) encontra-se na interseção de Gamma e o
círculo dado.


Seja MN um diâmetro de um círculo de centro O. Se A e B
são dois pontos neste diâmetro tais que M < A  < O < B < N ,
encontre o(s) ponto(s) P do círculo tal que o caminho APB
seja mínimo.


Sds,
Luís

-- 
Esta mensagem foi verificada pelo sistema de antivírus e
 acredita-se estar livre de perigo.



[obm-l] caminho mínimo (suite)

2018-05-23 Por tôpico Luís Lopes
Sauda,c~oes,


Começo nova mensagem pois àquelas que respondo não aparecem.


=


Boa tarde,


< Ponto do círculo ou da circunferência?
Circunferência.

< A ordenação que você menciona se refere ao ponto A estar entre
< M e O e o B estar entre O e N?
Isso.

Sds,
Luís

De: owner-ob...@mat.puc-rio.br <owner-ob...@mat.puc-rio.br> em nome de Pedro 
José <petroc...@gmail.com>
Enviado: quarta-feira, 23 de maio de 2018 19:49:26
Para: obm-l@mat.puc-rio.br
Assunto: [obm-l] Re: [obm-l] caminho mínimo

Boa tarde!
Ponto do círculo ou da circunferência?
A ordenação que você menciona se refere ao ponto A estar entre M e O e o B 
estar entre O e N?
Saudações,
PJMS

Em Qua, 23 de mai de 2018 15:18, Luís Lopes 
<qed_te...@hotmail.com<mailto:qed_te...@hotmail.com>> escreveu:
Sauda,c~oes,

Numa apostila do Curso Bahiense (Nº 13, Desenho
Geométrico) do Haroldo Manta (alguém aqui o conhece(u),
foi aluno dele ?) encontro o seguinte problema:

Seja MN um diâmetro de um círculo de centro O. Se A e B
são dois pontos neste diâmetro tais que M < A  < O < B < N ,
encontre o(s) ponto(s) P do círculo tal que o caminho APB
seja mínimo.

Luís

-- 
Esta mensagem foi verificada pelo sistema de antivírus e
 acredita-se estar livre de perigo.



[obm-l] caminho mínimo

2018-05-23 Por tôpico Luís Lopes
Sauda,c~oes,


Numa apostila do Curso Bahiense (Nº 13, Desenho

Geométrico) do Haroldo Manta (alguém aqui o conhece(u),

foi aluno dele ?) encontro o seguinte problema:


Seja MN um diâmetro de um círculo de centro O. Se A e B

são dois pontos neste diâmetro tais que M < A  < O < B < N ,

encontre o(s) ponto(s) P do círculo tal que o caminho APB

seja mínimo.


Luís


-- 
Esta mensagem foi verificada pelo sistema de antivírus e
 acredita-se estar livre de perigo.



mensagens não chegam (Era: [obm-l] Como calcular?)

2018-03-03 Por tôpico Luís Lopes
Sauda,c~oes,


Acho que a função "responder" está com problemas.

As mensagens que mando deste jeito não estão

aparecendo. E como tenho outro endereço de email,

vejo que algumas aparecem num e não aparecem no outro.


Decidi então criar uma totalmente nova, fora do encadeamento

das outras.


Espero que não esteja multiplicando o envio de mensagens

mas gostaria de ter certeza que estas duas últimas foram

realmente recebidas.


Abraços,

Luís


=

[obm-l] Como calcular?


Sauda,c~oes, oi Douglas,

Obrigado pelo link. Gostei muito do trabalho do Carlos Victor.

Aproveito para reenviar uma mensagem que mandei há alguns
dias e que acho que não chegou.
=


[obm-l] determine all pair of integers (x,y) such that

Sauda,c~oes,

Obrigado pelas respostas. Ideias bem legais
nas soluções.

Abraços,
Luís
**


-- 
Esta mensagem foi verificada pelo sistema de antivírus e
 acredita-se estar livre de perigo.



[obm-l] determine all pair of integers (x,y) such that

2018-02-24 Por tôpico Luís Lopes
1 + 2^x + 2^(2x+1) = y^2


Sauda,c~oes,


Recebi o problema acima de um outro grupo.

Como resolver ?


Abs,

Luís





-- 
Esta mensagem foi verificada pelo sistema de antivírus e
 acredita-se estar livre de perigo.



[obm-l] cadeia de logaritmos (suite)

2017-11-09 Por tôpico Luís Lopes
Eu acho que as mensagens que fazem parte deste assunto
(cadeia de logaritmos) não estão seguindo fazendo reply.
Mudo então o título do assunto e inicio um novo.

=
Sauda,c~oes, oi Bernardo,

Obrigado por responder.

Não sei provar a sua sugestão (e gostaria de ver a prova);
e pior, nem sei como usá-la.

Por sua mensagem e a do Tássio depreendo que o problema
faz sentido.

Retomo o que fiz.

Calculei a2 e encontrei 100 < a2 = 198 + log2 < 1000

Então 2 < a3=log(a2) < 3

0 < a4=log(a3) < 1

a5=log(a4) < 0

Logo a solução não é por aí.

Abraços,
Luís

-- 
Esta mensagem foi verificada pelo sistema de antivírus e
 acredita-se estar livre de perigo.



[obm-l] cadeia de logaritmos

2017-11-04 Por tôpico Luís Lopes
Sauda,c~oes,


Bom dia.


Não consegui resolver a questão abaixo. Como fazer ?


Abraços,

Luís



Considere o número N = ((100¹°°)¹°°)¨¨¹°° (ou seja, 100 elevado a 100, elevado 
a 100, ...), onde o número 100 aparece 100 vezes (incluindo a base). Seja a 
sequência definida como:

a1 = log N
a2 = log (a1)
a3 = log (a2)
...
a99 = log (a98)
a100 = log (a99)

Pode-se afirmar que a99 + a100 é igual a:

a) 102
b) 202
c) 102,3
d) 202,3
e) 2,3

Obs: se necessário, utilize log2 = 0,30.


-- 
Esta mensagem foi verificada pelo sistema de antivírus e
 acredita-se estar livre de perigo.



[obm-l] duas cordas paralelas

2017-10-24 Por tôpico Luís Lopes
Sauda,c~oes,


Sejam A e C dois pontos de um círculo Gamma=(O,R).

Construir duas cordas paralelas AB e CD tais que

AB+CD=2u, onde u é um comprimento dado.


Luís


-- 
Esta mensagem foi verificada pelo sistema de antivírus e
 acredita-se estar livre de perigo.



[obm-l] construção geométrica

2017-10-17 Por tôpico Luís Lopes
Sauda,c~oes,


Encontrei o problema abaixo num livro antigo de

Desenho Geométrico. Autor: Plácido Loriggio.


(MACK 57) Dados dois círculos tangentes de raios

iguais a 2cm e 5cm, respectivamente; construa uma

corda no círculo maior que tenha uma extremidade no

ponto de tangência e cujo segmento entre as duas

circunferências seja igual a 4cm.


Como fazer ?


Luís


-- 
Esta mensagem foi verificada pelo sistema de antivírus e
 acredita-se estar livre de perigo.



[obm-l] soma de tan^2

2017-09-16 Por tôpico Luís Lopes
Sauda,c~oes,


Bom dia.


Me mandaram a seguinte questão:


(1) Seja S = tan²(1º) + tan²(3º) + tan²(5º) + ... + tan²(89º), calcule o valor 
de S.


Como resolver ? Obrigado.


Abs,

Luís


-- 
Esta mensagem foi verificada pelo sistema de antivírus e
 acredita-se estar livre de perigo.



[obm-l] Cálculo de determinante

2017-03-01 Por tôpico Luís Lopes
Já mandei 2 ou 3 vezes esta mensagem para a lista.
Não sei por que ela não aparece. Tento novamente.

===
Oi, oi Douglas,

Sauda,c~oes,

Achei este problema legal e fiz uma busca por
"determinant of gcd matrix" no google.

Escolhi o link

http://math.stackexchange.com/questions/126/determinant-value-of-a-square-matrix-whose-each-entry-is-the-g-c-d-of-row-and-c

que me levou a

http://waset.org/publications/9996770/two-different-computing-methods-of-the-smith-arithmetic-determinant


< Obs: O resultado é MT bonito, uma potência de 2.
Verdade para n=1,2,….6. Fura para n=7.

Abs,
Luís

-- 
Esta mensagem foi verificada pelo sistema de antivírus e
 acredita-se estar livre de perigo.



[obm-l] {Disarmed} Enc: [EGML] Three circles in a triangle [1 Attachment]

2016-10-15 Por tôpico Luís Lopes
Sauda,c~oes,


Como construir (obter) o ponto D ?


A figura encontra-se no anexo (arquivo jpg).

Espero que ele a mostre.


Abs,

L.





[Attachment(s) from Antreas Hatzipolakis included below]

Nguyen Minh Tuan? wrote in a FB group (*):

Let ABC be a triangle and D a point such that the circles inscribed in the 
triangles ABD, BCD, and CAD pairwise touch each other. On lines BC, CA, AB, AD, 
BD, CD, denote the touching points by A1, B1, C1, A2, B2, C2, resp. Let lines 
B1C2 and B2C1 meet at E and let lines A1C2 and A2C1 meet at F . Show that the 
lines AF , BE and C1D are concurrent.

*

Let's replace E = B1C2 intersection B2C1 with Cb and F = A1C2 intersection A2C1 
with Ca.
Let C* be the point of concurrence of ACa , BCb and C1D.
Similarly A* and B*.

Are the triangles ABC, A*B*C* perspective?

APH

(*)

https://www.facebook.com/photo.php?fbid=1520551351291936=gm.1211999208873251=3


__._,_.___

Attachment(s) from Antreas Hatzipolakis | View attachments on the web 


1 of 1 Photo(s)

[$$1.jpg]
$$1.jpg



Reply via web 
post
*   Reply to sender 

   *   Reply to group 

*   Start a New 
Topic
*   Messages in this 
topic
 (1)

Check out the automatic photo album with 1 
photo(s)
 from this topic.
[$$1.jpg] 


[https://s.yimg.com/ru/static/images/yg/img/megaphone/1464031581_phpFA8bON]
Have you tried the highest rated email app?
With 4.5 stars in iTunes, the Yahoo Mail app is the highest rated email app on 
the market. What are you waiting for? Now you can access all your inboxes 
(Gmail, Outlook, AOL and more) in one place. Never delete an email again with 
1000GB of free cloud storage.

Visit Your 
Group

[Yahoo! 
Groups]
* Privacy * 
Unsubscribe * 
Terms of Use

.


__,_._,___

-- 
Esta mensagem foi verificada pelo sistema de antivírus e
 acredita-se estar livre de perigo.



Re: [obm-l] Re: [obm-l] nome de um quadrilátero

2016-08-10 Por tôpico Luís Lopes
Sauda,c~oes, oi Esdras,


Obrigado. Difícil imaginar isso pois rhombus

em inglês parece ser losango.


https://pt.wikipedia.org/wiki/Losango


Não me lembro de ter visto esse nome rombo.


Os livros didáticos usam esse nome para kite ?


Luís



De: owner-ob...@mat.puc-rio.br <owner-ob...@mat.puc-rio.br> em nome de Esdras 
Muniz <esdrasmunizm...@gmail.com>
Enviado: quarta-feira, 10 de agosto de 2016 21:13:26
Para: obm-l@mat.puc-rio.br
Assunto: [obm-l] Re: [obm-l] nome de um quadrilátero

Rombo.

Em 10 de agosto de 2016 17:43, Luís Lopes 
<qed_te...@hotmail.com<mailto:qed_te...@hotmail.com>> escreveu:

Sauda,c~oes,


Qual o nome em português para o

quadrilátero chamado de kite em inglês ?



https://en.wikipedia.org/wiki/Kite_(geometry)


Abs,

Luís


--
Esta mensagem foi verificada pelo sistema de antivírus e
acredita-se estar livre de perigo.



--
Esdras Muniz Mota
Mestrando em Matemática
Universidade Federal do Ceará



--
Esta mensagem foi verificada pelo sistema de antiv?rus e
acredita-se estar livre de perigo.

-- 
Esta mensagem foi verificada pelo sistema de antivírus e
 acredita-se estar livre de perigo.



[obm-l] nome de um quadrilátero

2016-08-10 Por tôpico Luís Lopes
Sauda,c~oes,


Qual o nome em português para o

quadrilátero chamado de kite em inglês ?



https://en.wikipedia.org/wiki/Kite_(geometry)


Abs,

Luís


-- 
Esta mensagem foi verificada pelo sistema de antivírus e
 acredita-se estar livre de perigo.



Re: [obm-l] soma binomial

2016-07-07 Por tôpico Luís Lopes
Sauda,c~oes, oi Anderson,


> Deve ter alguma forma de passar isso para uma função hipergeométrica

Deve ter. Tentei isso e só complicou.


> e ver se de fato tem solução fácil.

Ou melhor, uma solução esperta.

Pelo que sei do problema, deve ter. Vem do

Mathematical Reflections.


> Dei uma trapaceada, mas parece que o Wolfram Alpha não reconhece.

Ok. Nem pensei nisso. Mas acho que há programas capazes de

fornecer a forma fechada.


> Eu jogo diversos valores e isso tende a 1/3

Verdade. Fica S_n = 1/3 - ??


> - e o desejo de usar indução aumenta!

Verdade. A solução apresentada usa indução.

Mas acho nesse caso um pouco bastante artificial

pois o - ?? - acima veio do nada. A indução em si é fácil.


Na verdade comecei tentando S_n = \sum_{k=1}^n f(k)

com

f(k) = \frac{ k - 1 } { \binom{2k}{k} }

pois achei que era mais fácil com este f(k) do que com este aqui:

f(k) =  \frac{ 3k + 1 } { ( 2k + 1 ) \binom{2k}{k} } .


Neste deu pra calcular F(k) tal que F(k+1) - F(k) = \Delta F(k) = f(k)

e assim S_n = F(n+1) - F(1) = 1 - ?? .


Pro f(k) = \frac{ k - 1 } { \binom{2k}{k} } deve ter uma manipulação

binomial esperta pra obter o F(k) que não consigo ver.


Abs,

Luís



De: owner-ob...@mat.puc-rio.br <owner-ob...@mat.puc-rio.br> em nome de Anderson 
Torres <torres.anderson...@gmail.com>
Enviado: quinta-feira, 7 de julho de 2016 02:17:43
Para: obm-l@mat.puc-rio.br
Assunto: Re: [obm-l] soma binomial

Deve ter alguma forma de passar isso para uma função hipergeométrica e
ver se de fato tem solução fácil.

Dei uma trapaceada, mas parece que o Wolfram Alpha não reconhece. Eu
jogo diversos valores e isso tende a 1/3 - e o desejo de usar indução
aumenta!

Em 6 de julho de 2016 15:19, Luís Lopes <qed_te...@hotmail.com> escreveu:
> Sauda,c~oes,
>
>
> Alguém saberia como resolver (sem computador e indução) ?
>
> S_n = \sum_{k=1}^n f(k)
> com
> f(k) = \frac{ k-1 } { \binom{2k}{k} }.
>
> Abs,
> Luís
>
> --
> Esta mensagem foi verificada pelo sistema de antivírus e
> acredita-se estar livre de perigo.

--
Esta mensagem foi verificada pelo sistema de antiv?rus e
 acredita-se estar livre de perigo.


=
Instru??es para entrar na lista, sair da lista e usar a lista em
http://www.mat.puc-rio.br/~obmlistas/obm-l.html
Lista obm-l - Departamento de Matemática - 
PUC-Rio<http://www.mat.puc-rio.br/~obmlistas/obm-l.html>
www.mat.puc-rio.br
Lista obm-l. Existem pelo menos dois arquivos da lista obm-l. Um deles fica bem 
aqui, em http://www.mat.puc-rio.br/~obmlistas/obm-l.arquivo.html



=

-- 
Esta mensagem foi verificada pelo sistema de antivírus e
 acredita-se estar livre de perigo.



[obm-l] soma binomial

2016-07-06 Por tôpico Luís Lopes
Sauda,c~oes,

Alguém saberia como resolver (sem computador e indução) ?

S_n = \sum_{k=1}^n f(k)
com
f(k) = \frac{ k-1 } { \binom{2k}{k} }.

Abs,
Luís

-- 
Esta mensagem foi verificada pelo sistema de antivírus e
 acredita-se estar livre de perigo.



RE: [obm-l] funcao implicita e geogebra

2013-10-05 Por tôpico Luís Lopes
Sauda,c~oes, oi Bernardo, 

Bom, o que dizer? Muito obrigado, Bernardo!! 

  Continuo sem saber como calcular a equação que fornece
  os pontos extremos (max e min) da curva 
Agora sei. :) Pelo menos usando o WAlpha. 

 Se eu entendi o problema, você quer achar 
o(s) ponto(s) desta curva com a maior ordenada possível.
Exato. No intervalo 0 x10. 

 Agora, note que P(x(t),y(t)) = 0 em todos os pontos da curva, logo
 para todos os t da sua parametrização. Daí:
 dP/dx(x(t), y(t))*dx/dt + dP/dy(x(t), y(t))*dy/dt = 0. 
Comentário fundamental para a continuação. 

 Agora, você tem que achar as soluções do sistema {P(x,y) = 0,
 dP/dx(x,y) = 0}. No seu caso particular, dP/dx é de grau 2, logo você
 pode escrever y em função de x, e substituir na equação de grau 3...
No decorrer do pensamento vi que era por aí. Mas as contas 
se anunciavam pesadas. 

Os tais discriminantes são o caso particular do resolvente 
de um polinômio e de sua derivada. 
Bom, pode estar resumido mas estou satisfeito. 
Já sei o que são discriminantes neste contexto. 
E o melhor, o WA me dá eles. Coloquei o polinômio 
que dá a curva CC e ele me retornou a equação 
que já me haviam enviado. Daí calculei os pontos 
extremos da curva. Problema resolvido. 

O fator que sobra deve ser o polinômio de grau 4 que
passaram pra você.
Exato. 

Com isso o problema de construir o triângulo ABC dados 
A,a+b,h_a está resolvido e discutido.  O caso numérico 
estudado é cos A=11/14 e a+b=10. 

Se h_a  y_max  o problema não tem solução; 
se h_a=y_max um só triângulo satisfaz; 
se 0  h_a  y_max dois triângulos satisfazem. 

Deixo um problema com vocês: achar o lugar geométrico 
do vértice A dados o ângulo do vértice A e a diferença 
a-b. Seria a (nova) curva CC. 

Abs, 
Luís 






 Date: Sat, 5 Oct 2013 01:25:40 -0300
 Subject: Re: [obm-l] funcao implicita e geogebra
 From: bernardo...@gmail.com
 To: obm-l@mat.puc-rio.br
 
 2013/10/4 Luís Lopes qed_te...@hotmail.com:
  Sauda,c~oes,
 Oi Luís,
 
  Continuo sem saber como calcular a equação que fornece
  os pontos extremos (max e min) da curva mas talvez a
  teoria se encontre nos livros que tratam das Curvas Algébricas Planas.
 Exato, mas não necessariamente desta forma.
 
 Você tem uma equação implícita P(x,y) = 0, onde P é um polinômio (de
 grau 3) nas duas variáveis. Se eu entendi o problema, você quer achar
 o(s) ponto(s) desta curva com a maior ordenada possível.
 
 Para começar, pense que a curva definida pela equação P(x,y) = 0 é
 bonitinha (sem pontos duplos, sem singularidades, etc). Assim, imagine
 uma parametrização local da curva: (x(t), y(t)). Assim, pontos de
 máximo (e mínimo, ou de inflexão) são dados por dy/dt = 0.
 
 Agora, note que P(x(t),y(t)) = 0 em todos os pontos da curva, logo
 para todos os t da sua parametrização. Daí:
 dP/dx(x(t), y(t))*dx/dt + dP/dy(x(t), y(t))*dy/dt = 0.
 
 Se dy/dt = 0, temos então que dP/dx(x(t), y(t)) * dx/dt = 0. Como a
 curva é lisa (e sem singularidades, pontos duplos, etc, etc), dx/dt
 não pode ser = 0. Daí, os pontos de máximo também satisfazem uma
 equação suplementar, dP/dx(x,y) = 0.
 
 Agora, você tem que achar as soluções do sistema {P(x,y) = 0,
 dP/dx(x,y) = 0}. No seu caso particular, dP/dx é de grau 2, logo você
 pode escrever y em função de x, e substituir na equação de grau 3...
 
 Mas, em geral, existe uma teoria para achar as soluções de sistemas de
 equações simultâneas, que são os resolventes, cf
 http://en.wikipedia.org/wiki/Resultant e
 http://en.wikipedia.org/wiki/Sylvester_matrix, em particular a parte
 de aplicações a interseções do resultante. Essencialmente, eles
 substituem uma equação na outra, de forma algorítmica, e dão
 diretamente a equação que todas (e não metade, como seria o caso se
 você pegasse cada uma das soluções da eq do segundo grau) as abscissas
 possíveis y satisfazem. Os tais discriminantes são o caso particular
 do resolvente de um polinômio e de sua derivada. Note que isso envolve
 considerar P(x,y) como um polinômio em x cujos coeficientes são
 polinômios em y. Os livros de Curvas Algébricas em geral, vão falar
 deste parágrafo apenas (provar que a Resultante realmente faz a
 eliminação das variáveis, como calcular, como que os discriminantes
 têm a ver com resultantes).
 
 Exemplinho: Seja P(x,y) = (x^2 + x y^2 + x y + 2y + 2) = x^2 + (y^2 +
 y)*x + (2y + 1). Chame a = 1, b = (y^2 + y), c = (2y + 1). Nesse caso,
 o discriminante é o usual, ou seja, b^2 - 4*a*c = (y^4 + 2y^3 + y^2)
 - 8y - 4. As raízes disso dão os y máximos e mínimos locais.
 Dá pra ver tudo isso com o WolframAlpha
 
 http://www.wolframalpha.com/input/?i=+x^2+%2B+%28y^2+%2B+y%29*x+%2B+%282y+%2B+1%29+%3D+0
 http://www.wolframalpha.com/input/?i=%28y^4+%2B+2y^3+%2B+y^2%29+-+8y+-+4+%3D+0
 http://www.wolframalpha.com/input/?i=discriminant%28+x^2+%2B+%28y^2+%2B+y%29*x+%2B+%282y+%2B+1%29%2C+x%29
 
 
 Voltando ao seu problema, o WA dá o discriminante:
 http://www.wolframalpha.com/input/?i=discriminant%28%28-s^2y%2Bx^2y%2By^3%29Cos[A]%2B%28-s^2x%2B2sx^2-x^3

RE: [obm-l] funcao implicita e geogebra

2013-10-04 Por tôpico Luís Lopes
Sauda,c~oes, 

Problema resolvido mas desconheço a teoria que 
fornece a equação para calcular o máximo e mínimo 
da curva. 
 
Máximo e mínimo valor de y de la curva: Hay que resolver en y la ecuación: 



-s^4 + 10 s^2 y^2 + 2 y^4 + s^2 (s^2 - 12 y^2)Cos[2A] + (-6 s^3 y + 8 s 
y^3)Sin[2A] = 0

Foi falado num determinante,  sem maiores detalhes. 

E numa mensagem recente daqui (problema de tangência 

numa elipse) falou-se de um determinante também. 
Deve-se tratar da mesma coisa. 



Sds, 
Luís 


From: qed_te...@hotmail.com
To: obm-l@mat.puc-rio.br
Subject: [obm-l] funcao implicita e geogebra
Date: Thu, 3 Oct 2013 21:30:41 +







Sauda,c~oes, 

Não conheço muito do Geogebra e talvez alguém aqui 
possa me ajudar. 

O que segue é uma investigação sobre o problema de 
construir o triângulo ABC dados A,a+b,h_a. Algebricamente 
somente pois temos uma cúbica nos cálculos. 

Consegui descobrir que um lugar geométrico para o vértice A 
é dado pela cúbica 

===
La ecuación del lugar geométrico es:

(-s^2y+x^2y+y^3)Cos[A]+(-s^2x+2sx^2-x^3+2sy^2-xy^2)Sin[A]=0 
===
com s=a+b. 

Gostaria de calcular o valor máximo de y (que aqui representa a 
altura do triângulo) e assim ter somente uma solução.

No que segue s=10, Cos[A]=11/14 e Sin[A]=5sqrt(3)/14. 

Fui pro Geogebra e entrei os pontos B=(0,0) e A'=(10,0), bem como 
a função implícita do locus. 

(-10² y + x² y + y³) 11 / 14 + (-10² x + 2 (10) x² - x³ + 2 (10) y² - x y²) 5 
sqrt(3) / 14 = 0




Em seguida tracei a reta h_a=y=4.5 e obtive as duas interseções 
(vértices A) que interessam: A1 = (-0.38975, 4.5)


e A2 = (6.73216, 4.5)




. 
Daí C1 = (3.83061, 0)


e C2 = (5.26770, 0). Ou seja, obtive dois triângulos 
satisfazendo as condições. 

Até aqui tudo bem. Agora quero saber o valor máximo de y ou o ponto 
onde a derivada de 





(-10² y + x² y + y³) 11 / 14 + (-10² x + 2 (10) x² - x³ + 2 (10) y² - x y²) 5 
sqrt(3) / 14 = 0




se anula. Chamei esta equação de CC (para closed curve).

Voltei pro Geogebra e tentei usar o comando implicitderivative e ele me 
retornou 



CC'(x, y) = (15sqrt(3) x² - 200sqrt(3) x + 5sqrt(3) y² + 500sqrt(3) - 22x y) / 
(-10 sqrt(3) x y + 200sqrt(3) y + 11x² + 33y² - 1100)



Não sei bem o que estou fazendo agora. Ou mesmo como continuar sem o 
Geogebra. Alguém pode me explicar, continuar a investigação 
ou me dar o ponto de máximo? 

Obrigado. 

Sds, 
Luís 


  
--

Esta mensagem foi verificada pelo sistema de antivírus e 

 acredita-se estar livre de perigo.   
-- 
Esta mensagem foi verificada pelo sistema de antivírus e
 acredita-se estar livre de perigo.



RE: [obm-l] funcao implicita e geogebra

2013-10-04 Por tôpico Luís Lopes
Sauda,c~oes, 

Li errado. Não é determinante e sim discriminante. 

Continuo sem saber como calcular a equação que fornece 
os pontos extremos (max e min) da curva mas talvez a 
teoria se encontre nos livros que tratam das Curvas Algébricas Planas. 

Sds, 
Luís 


From: qed_te...@hotmail.com
To: obm-l@mat.puc-rio.br
Subject: RE: [obm-l] funcao implicita e geogebra
Date: Fri, 4 Oct 2013 20:02:40 +




Sauda,c~oes, 

Problema resolvido mas desconheço a teoria que 
fornece a equação para calcular o máximo e mínimo 
da curva. 
 
Máximo e mínimo valor de y de la curva: Hay que resolver en y la ecuación: 



-s^4 + 10 s^2 y^2 + 2 y^4 + s^2 (s^2 - 12 y^2)Cos[2A] + (-6 s^3 y + 8 s 
y^3)Sin[2A] = 0

Foi falado num determinante,  sem maiores detalhes. 

E numa mensagem recente daqui (problema de tangência 
numa elipse) falou-se de um determinante também. 
Deve-se tratar da mesma coisa. 

Sds, 
Luís 


From: qed_te...@hotmail.com
To: obm-l@mat.puc-rio.br
Subject: [obm-l] funcao implicita e geogebra
Date: Thu, 3 Oct 2013 21:30:41 +







Sauda,c~oes, 

Não conheço muito do Geogebra e talvez alguém aqui 
possa me ajudar. 

O que segue é uma investigação sobre o problema de 
construir o triângulo ABC dados A,a+b,h_a. Algebricamente 
somente pois temos uma cúbica nos cálculos. 

Consegui descobrir que um lugar geométrico para o vértice A 
é dado pela cúbica 

===
La ecuación del lugar geométrico es:

(-s^2y+x^2y+y^3)Cos[A]+(-s^2x+2sx^2-x^3+2sy^2-xy^2)Sin[A]=0 
===
com s=a+b. 

Gostaria de calcular o valor máximo de y (que aqui representa a 
altura do triângulo) e assim ter somente uma solução.

No que segue s=10, Cos[A]=11/14 e Sin[A]=5sqrt(3)/14. 

Fui pro Geogebra e entrei os pontos B=(0,0) e A'=(10,0), bem como 
a função implícita do locus. 

(-10² y + x² y + y³) 11 / 14 + (-10² x + 2 (10) x² - x³ + 2 (10) y² - x y²) 5 
sqrt(3) / 14 = 0




Em seguida tracei a reta h_a=y=4.5 e obtive as duas interseções 
(vértices A) que interessam: A1 = (-0.38975, 4.5)


e A2 = (6.73216, 4.5)




. 
Daí C1 = (3.83061, 0)


e C2 = (5.26770, 0). Ou seja, obtive dois triângulos 
satisfazendo as condições. 

Até aqui tudo bem. Agora quero saber o valor máximo de y ou o ponto 
onde a derivada de 





(-10² y + x² y + y³) 11 / 14 + (-10² x + 2 (10) x² - x³ + 2 (10) y² - x y²) 5 
sqrt(3) / 14 = 0




se anula. Chamei esta equação de CC (para closed curve).

Voltei pro Geogebra e tentei usar o comando implicitderivative e ele me 
retornou 



CC'(x, y) = (15sqrt(3) x² - 200sqrt(3) x + 5sqrt(3) y² + 500sqrt(3) - 22x y) / 
(-10 sqrt(3) x y + 200sqrt(3) y + 11x² + 33y² - 1100)



Não sei bem o que estou fazendo agora. Ou mesmo como continuar sem o 
Geogebra. Alguém pode me explicar, continuar a investigação 
ou me dar o ponto de máximo? 

Obrigado. 

Sds, 
Luís 


  
--

Esta mensagem foi verificada pelo sistema de antivírus e 

 acredita-se estar livre de perigo.   
--

Esta mensagem foi verificada pelo sistema de antivírus e 

 acredita-se estar livre de perigo.   
-- 
Esta mensagem foi verificada pelo sistema de antivírus e
 acredita-se estar livre de perigo.



[obm-l] funcao implicita e geogebra

2013-10-03 Por tôpico Luís Lopes



Sauda,c~oes, 

Não conheço muito do Geogebra e talvez alguém aqui 
possa me ajudar. 

O que segue é uma investigação sobre o problema de 
construir o triângulo ABC dados A,a+b,h_a. Algebricamente 
somente pois temos uma cúbica nos cálculos. 

Consegui descobrir que um lugar geométrico para o vértice A 
é dado pela cúbica 

===
La ecuación del lugar geométrico es:

(-s^2y+x^2y+y^3)Cos[A]+(-s^2x+2sx^2-x^3+2sy^2-xy^2)Sin[A]=0 
===
com s=a+b. 

Gostaria de calcular o valor máximo de y (que aqui representa a 
altura do triângulo) e assim ter somente uma solução.

No que segue s=10, Cos[A]=11/14 e Sin[A]=5sqrt(3)/14. 

Fui pro Geogebra e entrei os pontos B=(0,0) e A'=(10,0), bem como 
a função implícita do locus. 

(-10² y + x² y + y³) 11 / 14 + (-10² x + 2 (10) x² - x³ + 2 (10) y² - x y²) 5 
sqrt(3) / 14 = 0




Em seguida tracei a reta h_a=y=4.5 e obtive as duas interseções 
(vértices A) que interessam: A1 = (-0.38975, 4.5)


e A2 = (6.73216, 4.5)




. 
Daí C1 = (3.83061, 0)


e C2 = (5.26770, 0). Ou seja, obtive dois triângulos 
satisfazendo as condições. 

Até aqui tudo bem. Agora quero saber o valor máximo de y ou o ponto 
onde a derivada de 





(-10² y + x² y + y³) 11 / 14 + (-10² x + 2 (10) x² - x³ + 2 (10) y² - x y²) 5 
sqrt(3) / 14 = 0




se anula. Chamei esta equação de CC (para closed curve).

Voltei pro Geogebra e tentei usar o comando implicitderivative e ele me 
retornou 



CC'(x, y) = (15sqrt(3) x² - 200sqrt(3) x + 5sqrt(3) y² + 500sqrt(3) - 22x y) / 
(-10 sqrt(3) x y + 200sqrt(3) y + 11x² + 33y² - 1100)



Não sei bem o que estou fazendo agora. Ou mesmo como continuar sem o 
Geogebra. Alguém pode me explicar, continuar a investigação 
ou me dar o ponto de máximo? 

Obrigado. 

Sds, 
Luís 


  
-- 
Esta mensagem foi verificada pelo sistema de antivírus e
 acredita-se estar livre de perigo.



[obm-l] RE: [obm-l] Re: [obm-l] RE: [obm-l] Construção Geométrica (triângulos) ITA 1989

2013-07-01 Por tôpico Luís Lopes
Sauda,c~oes, oi Sergio, 
No google triangle construction given H_a,W_a,O aparecem outras soluções e 
comentários. 
Qual a fonte da sua construção ? 

Abs, Luis 

Date: Mon, 1 Jul 2013 09:58:53 -0300
Subject: [obm-l] Re: [obm-l] RE: [obm-l] Construção Geométrica (triângulos) ITA 
1989
From: sergi...@smt.ufrj.br
To: obm-l@mat.puc-rio.br

Caros,
Complementando entao a resposta do Luís Lopes,aqui vai a solução do problema:


ANÁLISE DO PROBLEMA:
Seja M a projeção de O na reta suporte de DH.Supondo a solução do problema 
conhecida,seja M´ a interseção de OM com a circunferência circunscrita.


Por uma análise angular simples é possível concluir queAOM' = (A + 2B) [ou (A 
+ 2C)], de modo que OAM' = OM'A = (C-B)/2[ou (B-C)/2]. Assim, AM' é a própria 
bissetriz interna do ângulo A
no triângulo desejado ABC, e, por isso mesmo, D pertence a AM'.
No triângulo AOM', com OA = OM', seja P1a altura do vértice O relativa ao lado 
AM'. Assim, temos duas propriedades
que nos permitem determinar o ponto P1:
(i) Como AOM' é isósceles, OP1 é perpendicular a AM' (e a AD).Assim, OP1D = 90 
graus, de modo que P1 pertence à circunferência de diâmetro OD.

(ii) Como AOM' é isósceles, P1 é o ponto médio de AM'. Como OM' é paralelaa AH 
(ambas são perpendiculares à reta suporte de DH), P1pertence à reta paralela a 
essas duas retas (OM' e AH) passando pelo ponto médio
de HM.

CONSTRUÇÂO
(i) trace a circunferência C1 de centro O1 e raio OO1, onde O1 é o ponto médio 
de OD,determinando a interseção M (ponto médio do lado BC) sobre a reta suporte 
de DH.
(ii) trace a perpencidular p à reta suporte de DH pelo ponto médio P de 
MH,determinando sobre C1 a(s) interseção(ões) P1 (e P2).(iii) prolongue DP1, 
determinando o vértice A sobre a perpendicular a DH por H.
(o prolongamento de DP2 gera uma outra solução para o vértice A).(iv) trace a 
circunferência de centro O e raio OA circunscrita ao triângulo,determinando os 
outros dois vértices B e C sobre a reta suporte de DH.

OBS 1: É possível ter 0/1/2 solução(ões) para o vértice A,dependendo se a reta 
p não-intercepta/tangencia/é-secante a C1.
OBS 2: Os vértices B e C podem ser intercambiados.

Abracos,sergio

2013/6/26 Luís Lopes qed_te...@hotmail.com




Sauda,c~oes, oi Sergio, 
Sim, continuo na lista. 
Caiu no ITA, foi? Bom saber. 
Gosto mesmo destes problemas. Vou mandar em seguida mais 
um, que acabo de conhecer. Problema (presente) de grego. 
===Eu não consegui, mas obtive a solução na internet(a qual envio numa próxima 
mensagem).
===Fico curioso. Conseguir como? Com o Google?? E e e ??? 
Para construir o triângulo, precisamos conhecer um resultado fundamental: a 
bissetriz ASa é bissetriz também do ângulo 
HaAO. 
Outro fato, esse elementar: a reta (A , Ha) é perpendicular â reta (Ha , Sa). 
Ultima dica: pense num circulo e numa reta espertos . 

Valeu Sergio pelo problema. 
Abs, Luis 

Date: Wed, 26 Jun 2013 08:01:02 -0300
Subject: [obm-l] Construção Geométrica (triângulos) ITA 1989

From: sergi...@smt.ufrj.br
To: obm-l@mat.puc-rio.br

Essa é em homenagem ao Luís Lopes e ao E. Wagner
(não sei se ainda acompanham a lista):
Construa o triângulo ABC dados em posição:. o pé Ha da altura do vértice A em 
relação ao lado BC.
. a interseção Sa da bissetriz do ângulo A com o lado BC.. o circuncentro O 
do triângulo.
Eu não consegui, mas obtive a solução na internet
(a qual envio numa próxima mensagem).
Abraço,sergio


--

Esta mensagem foi verificada pelo sistema de antivírus e 

 acredita-se estar livre de perigo.   
--

Esta mensagem foi verificada pelo sistema de antivírus e 

 acredita-se estar livre de perigo.





--

Esta mensagem foi verificada pelo sistema de antivírus e 

 acredita-se estar livre de perigo.   
-- 
Esta mensagem foi verificada pelo sistema de antivírus e
 acredita-se estar livre de perigo.



[obm-l] presente de grego 2

2013-07-01 Por tôpico Luís Lopes
Sauda,c~oes, 
Da mesma lista do anterior. 

[APH]In a triangle are given: a, A and the ratio |b-c| / h_a = m/n  (where 
h_a is the altitude from A) Prove that it has an Euclidean construction. 
(Ref.: Parartema, Nov. 1939, p.393)[Luis]: Again, I have no idea.  May I have 
a hint ?  Thanks. Dear Luis,Whenever we have an angle [so sin(of_that_angle) = 
sin(of_the_sum_of_the_other_two)] a good idea is to express the sin or cosof 
the difference of the other angles by the data of theproblem.If the equation we 
get is of = 2 degree it is possibleto have an euclidean construction.But how 
to make that construction geometrically is another story
Alguém saberia construir o triângulo dados a, A and the ratio |b-c| / h_a = m/n 
 ? 
Ou expressar sin(B-C) em função destes dados ? 
Abs, Luis 
  
-- 
Esta mensagem foi verificada pelo sistema de antivírus e
 acredita-se estar livre de perigo.



[obm-l] RE: [obm-l] Construção Geométrica (triângulos) ITA 1989

2013-06-26 Por tôpico Luís Lopes
Sauda,c~oes, oi Sergio, 
Sim, continuo na lista. 
Caiu no ITA, foi? Bom saber. 
Gosto mesmo destes problemas. Vou mandar em seguida mais um, que acabo de 
conhecer. Problema (presente) de grego. 
===Eu não consegui, mas obtive a solução na internet(a qual envio numa próxima 
mensagem).===Fico curioso. Conseguir como? Com o Google?? E e e ??? 
Para construir o triângulo, precisamos conhecer um resultado fundamental: a 
bissetriz ASa é bissetriz também do ângulo HaAO. 
Outro fato, esse elementar: a reta (A , Ha) é perpendicular â reta (Ha , Sa). 
Ultima dica: pense num circulo e numa reta espertos . 
Valeu Sergio pelo problema. 
Abs, Luis 

Date: Wed, 26 Jun 2013 08:01:02 -0300
Subject: [obm-l] Construção Geométrica (triângulos) ITA 1989
From: sergi...@smt.ufrj.br
To: obm-l@mat.puc-rio.br

Essa é em homenagem ao Luís Lopes e ao E. Wagner(não sei se ainda acompanham a 
lista):
Construa o triângulo ABC dados em posição:. o pé Ha da altura do vértice A em 
relação ao lado BC.
. a interseção Sa da bissetriz do ângulo A com o lado BC.. o circuncentro O 
do triângulo.
Eu não consegui, mas obtive a solução na internet
(a qual envio numa próxima mensagem).
Abraço,sergio


--

Esta mensagem foi verificada pelo sistema de antivírus e 

 acredita-se estar livre de perigo.   
-- 
Esta mensagem foi verificada pelo sistema de antivírus e
 acredita-se estar livre de perigo.



[obm-l] Presente de Grego

2013-06-26 Por tôpico Luís Lopes
Sauda,c~oes, 
Este problema apareceu nesta semana, numa outra lista da qual faço parte. 
To construct triangle if are given:1. the length of the altitude from A2. the 
length of the int. bisector of A3. the length of the orthogonal projection of 
the side BCon the external bisector of AReference: It is an old problem I found 
in a Greek math. magazine(Euclid, June 1969, p. 405)APH
Ou seja, construir ABC dados ha, da, a'. 
Dica facil: construir o triângulo AHaDa e o ponto Ea. Da e Ea s~ao os pés das 
bissetrizes interna e externa. 
Dica dificil: M é o ponto médio de AEa. N é o ponto médio de B'C'=a'. 
Abs, Luis 
  
-- 
Esta mensagem foi verificada pelo sistema de antivírus e
 acredita-se estar livre de perigo.



[obm-l] equacao exponencial

2013-05-14 Por tôpico Luís Lopes

Sauda,c~oes, 

Pediram a minha ajuda no problema abaixo. 
Se sair truncado para alguns, o problema é: 

O número de pontos comuns aos gráficos das funções definidas por $y=e^x$ e $y= 
- \ln |x|$,  
$x\neq0$, é:
Como vocês sempre têm uns comentários espertos que me escapam, 
aguardo suas respostas. O email veio com a resposta, que cortei. 

Agora um pedido meu: gostaria de ter as respostas, não somente o número delas. 
Um Maple qualquer dá isso. Obrigado. 

Abs, 
Luís 

Caro Luis,
Gostaria de sua ajuda para a seguinte questão:
O número de pontos comuns aos gráficos das funções definidas por e , , é:

a) .
b) .
c) .
d) .
e) nenhuma das anteriores.
 RESPOSTA: 
??

[obm-l] A,a,h_a construir triangulo

2013-05-12 Por tôpico Luís Lopes
Sauda,c~oes, 

O problema de construir o triângulo ABC dados A,a,h_a 
é bem conhecido. Aparece como a,h_a,R no livro do Wagner 
de Construções Geométricas e é resolvido pelo Sérgio Lima Netto 
É o problema 28 do Capítulo 1. 

A construção que conheço e a mesma que vejo sempre nos livros 
usa o arco capaz de A sobre o segmento BC=a. O que vale dizer, 
o círculo circunscrito e seu raio R. Talvez por ser a mais  
natural??, evidente?? 

Isso se conhecemos e sabemos construir o arco capaz. Mas NÃO sei. 

Pensei então numa outra construção. Começo construindo o ângulo A. 
Assim B e C pertencem às retas que formam o ângulo A. E já sei também 
um lugar geométrico para H_a, pé da altura. Falta construir a reta 
suporte do lado a. Isso eu não sei. 

Como terminar a construção? 

Abs, 
Luís 


  

[obm-l] soma trigonometrica

2012-12-10 Por tôpico Luís Lopes







Sauda,c~oes, 

No último número da Eureka  


http://www.obm.org.br/export/sites/default/revista_eureka/docs/Eureka35.pdf 
www.obm.org.br/export/sites/default/revista_eureka/docs/Eureka35.pdf


encontrei na página 54  o problema 147. 

O problema é: mostrar que para n\geq 2 

S_n = \sum_{k=0}^{n-2} \frac{(-1)^k}{\sin\Bigl(\frac{(2k+1)\pi}{4n-2}\Bigr)} = 
n + \frac{(-1)^n - 1}{2} 

n=2  S_2=2

n=3  S_3=\csc(\pi/10) - \csc(3\pi/10) = 2. 

n=4  S_4=\csc(\pi/14) - \csc(3\pi/14) +  \csc(5\pi/14) = 4. 

Alguém (da Eureka?) poderia falar alguma coisa dessa fórmula? Tipo  

1) referência de onde foi tirada. 
2) outra demonstração. Seria possível por indução? Por recorrência? 
3) seria um caso particular de uma fórmula mais geral? 

Felicito o Michel Martins (autor da demonstração apresentada) pela 
demonstração. 

Abraços, 
Luís 



  

RE: [obm-l] Calcular o mdc (333...3, 333...3)

2012-12-09 Por tôpico Luís Lopes

Sauda,c~oes, 

Então quero calcular mdc entre (x^10 - 1) e (x^8 - 1), onde x=10^10.
Esta questão foi respondida pelo Buffara em 30/3/2007. 
Seja mdc(m,n)=d. Então mdc(x^n-1,x^m-1)=x^d-1. 

Off topic. 
Fiquei há uns dez dias sem receber as msgs da lista, mesmo as que eu 
mandava. Mas numa outra conta de email elas apareciam. Ontem recebi 
muitas msgs, acho que todas que havia ficado sem receber. E algumas 
(como as do Artur e Eduardo e outras) vinham com uma advertência do 
filtro do hotmail como msgs suspeitas. E vejo que o Artur e Eduardo enviam 
as mesmas msgs mais de uma vez. Devem estar com o mesmo problema. 
Suspeito que o site da obm-l esteja contaminado. 

Abraços, 
Luís 



From: marconeborge...@hotmail.com
To: obm-l@mat.puc-rio.br
Subject: RE: [obm-l] Calcular o mdc (333...3, 333...3)
Date: Sat, 8 Dec 2012 22:42:08 +





Muito legal!
 
Date: Wed, 5 Dec 2012 02:48:28 -0200
Subject: Re: [obm-l] Calcular o mdc (333...3, 333...3)
From: wgapetre...@gmail.com
To: obm-l@mat.puc-rio.br

O 1o numero é (10^100 - 1)/3, enquanto o 2o é (10^80 - 1)/3. Obviamente eu 
posso ignorar esse 1/3 aí, e depois dividir a resposta que eu achar por 3.
Então quero calcular mdc entre (x^10 - 1) e (x^8 - 1), onde x=10^10.

Então eu percebo que x^2 - 1 divide ambos (se eu não percebesse, eu sempre 
poderia fazer a divisão euclidiana deles). Dividindo tudo por x^2 - 1 fica:
p(x) = x^8 + x^6 + x^4 + x^2 + 1 e q(x) = x^6 + x^4 + x^2 + 1. Então eu percebo 
que p(x) - x^2*q(x) = 1 (novamente se eu não percebesse eu faria a divisão...). 
Então acabou pq se alguém divide p(x) e q(x) para algum x, então também divide 
1. Logo esses caras são primos entre si. Assim o mdc original fica (x^2-1)/3 = 
10^20 - 1



2012/12/4 Pedro Chaves brped...@hotmail.com





Colegas da Lista,

Como calcular o mdc (a, b) , sendo a = 333...3  (100 dígitos iguais a 3) e b = 
333...3 (80 dígitos iguais a 3)?


Abraços do pedro Chaves

_-




  

[obm-l] RE: PROBLEMA

2012-12-05 Por tôpico Luís Lopes

Sauda,c~oes, 

Não tenho recebido e conseguido mandar msgs para a lista. 

Esta vai como teste. Se receberem, continuem a ler. 

Luís 

From: qed_te...@hotmail.com
To: obm-l@mat.puc-rio.br
Subject: FW: PROBLEMA
Date: Mon, 3 Dec 2012 12:44:01 +





Sauda,c~oes, 

Mais uma vez, peço a ajuda de vocês para uma resposta 
mais completa e interessante. 

Espero que o anexo passe. Se não, alguém poderia colocá-lo 
disponível em algum site? 


Subject: PROBLEMA
Date: Mon, 3 Dec 2012 08:17:24 -0300








Prezado Luís, 



Gostaria de sua ajuda para o problema(conforme 
arquivo PDF anexo). 
Se possível, gostaria que justificasse porque os demais 
itens são verdadeiros. 


Abraços, 

FERNANDO FORTALEZA-CE. 

PS: E eu gostaria de saber também por que o item falso é falso. 
Obrigado. 

Abraços, 
Luís 



  

[obm-l] PROBLEMA

2012-12-05 Por tôpico Luís Lopes

Sauda,c~oes, 

Mais uma tentativa de ver esta mensagem chegar na lista. 

O Bernardo já respondeu numa troca de emails particular. 

Esta mensagem deve ser lida de baixo para cima. 

Abraços, 
Luís 



O arquivo está em 
http://www.escolademestres.com/download/questao_luis_lopes_20121205.pdf 

 Date: Wed, 5 Dec 2012 09:59:41 -0500
 Subject: Re: [obm-l] RE: PROBLEMA
 From: bernardo...@gmail.com
 To: qed_te...@hotmail.com
 
 2012/12/5 Luís Lopes qed_te...@hotmail.com:
  Oi Bernardo,
 
  Não consigo me comunicar com a lista. Mas pelo jeito a
  lista (você) recebe(u) minhas msgs.
 Estranho.
 
  Tenho recebido somente algumas msgs da lista. Num outro
  email (outra conta) acho que recebo todos. Pode ser problema
  com o hotmail também, sei lá.
 Pode ser...
 
  Tudo começou com o email abaixo, que mandei pra lista:
 Pode ser uma questão de PDFs. Sei lá. Pode ser mesmo esse PDF em particular.
 
 Para responder a pergunta: todas as afirmações são verdadeiras, é
 claro. O problema da d é que a projeção estereográfica envia
 círculos passando pelo polo Norte em retas do plano + ponto no
 infinito. Mas como f está definida apenas na esfera - polo Norte,
 nenhuma dessas circunferências serve, porque f não aplica o polo norte
 em lugar nenhum. Logo, as circunferências que você *pode* usar são
 realmente mandadas em círculos do plano.
 
 Ah, óbvio que isso é um problema de geometria complexa, mas fica muito
 mais legal retirar todo o contexto e fazer uma questão absurda de
 vestibular. Pena, né.
 -- 
 Bernardo Freitas Paulo da Costa

From: qed_te...@hotmail.com
To: obm-l@mat.puc-rio.br
Subject: FW: PROBLEMA
Date: Mon, 3 Dec 2012 12:44:01 +





Sauda,c~oes, 

Mais uma vez, peço a ajuda de vocês para uma resposta 
mais completa e interessante. 

Espero que o anexo passe. Se não, alguém poderia colocá-lo 
disponível em algum site? 


Subject: PROBLEMA
Date: Mon, 3 Dec 2012 08:17:24 -0300








Prezado Luís, 



Gostaria de sua ajuda para o problema(conforme 
arquivo PDF anexo). 
Se possível, gostaria que justificasse porque os demais 
itens são verdadeiros. 


Abraços, 

FERNANDO FORTALEZA-CE. 

PS: E eu gostaria de saber também por que o item falso é falso. 
Obrigado. 

Abraços, 
Luís 

  

[obm-l] series telescopicas

2012-09-24 Por tôpico Luís Lopes

Sauda,c~oes, 

Recebi o seguinte email: 


Boa tarde! Caro Prof. Luís Lopes, estou interessado em saber mais sobre
 as séries telescópicas, sendo mais especifico sobre a origem do termo 
telescópica, o porquê desse nome e como ele surgiu para definir esse 
tipo de série, desde já agradeço a atenção e estarei aguardando 
ansiosamente uma resposta. Obrigado!


O porquê eu até acho que sei: aquele tubo da luneta se espicha e 
se recolhe sobre si mesmo. Daí a imagem com as séries telescópicas. 

Mas origem do termo, como e quando surgiu . ?? 

[]'s 
Luís 

  

[obm-l] RE: [obm-l] Re: [obm-l] Geometria(Construção(2))

2012-09-12 Por tôpico Luís Lopes

Sauda,c~oes, oi Ralph, 

Gostei da sua construção do triângulo. Eu começaria 
traçando o Â. Depois a bissetriz etc. Mas a sua 
construção é melhor. 

No quadrilátero APIQ o PÎQ = 180 - Â. Então o 
tamanho do arco PQ não seria 180 - Â ?? 

Para que a construção funcione, precisamos que Q esteja entre A e C, isto é, 
que br.sin(Â/2)AQ = r.cot(Â/2). Logo, br.cot(Â/2) 

[]'s 
Luís 



Date: Tue, 11 Sep 2012 13:47:40 -0300
Subject: [obm-l] Re: [obm-l] Geometria(Construção(2))
From: ralp...@gmail.com
To: obm-l@mat.puc-rio.br

1) Considere o círculo de diâmetro P1P2. Ele contém o vértice A do quadrado... 
Mas, melhor ainda, pense na diagonal AC! Como ela é a bissetriz do ângulo 
P1AP2, então ela passa pelo ponto D1, médio do arco P1P2 daquele círculo (veja 
figura anexa, viva Geogebra!), que é DETERMINADO A PARTIR DE P1 e P2!

Analogamente, você pode encontrar D3, médio do arco P3P4 do círculo de diâmetro 
P3P4. Como D1D3 será a diagonal do quadrado, você pode intersectar esta 
diagonal com os círculos para achar A e C.

Agora repita para B e D, e acabou!
(Se alguém quiser o Geogebra da construção, mando por E-mail -- acho que a 
lista não aceitaria o anexo)
2) Este é bem mais simples:
i) Desenhe o círculo inscrito, marque nele um arco PQ de tamanho 180-2A.ii) 
Trace as tangentes ao círculo por P e Q, intersecte-as, este é o ponto A (note 
que PÂQ=Â, de fato).iii) Agora é só marcar b (a partir de A) em cima da reta AQ 
para achar o ponto C...
iv)... e traçar a tangente ao círculo por C para achar o ponto B (sobre AP).
Para que a construção funcione, precisamos que Q esteja entre A e C, isto é, 
que br.sin(Â/2)

Abraço,   Ralph
P.S.: Note que há um outro arco P1P2! O que aconteceria se a gente escolhesse 
D1 como médio desse OUTRO arco, assim como D3, D2 e D4? :) :) :)

2012/9/10 marcone augusto araújo borges marconeborge...@hotmail.com








1)Os pontos P1,P2,P3,P4 pertencem aos lados consecutivos de um quadrado 
ABCD.Construa com régua e compasso o quadrado.Justifique sua construção.
 
 
 .P1
 
   .P2

 
 
 
.P3  
 

 .P4 
 
 2) Construa o triangulo ABC conhecendo o angulo A,o lado b e o raio r do 
círculo inscrito.Justifique.   

  

  

[obm-l] RE: [obm-l] Re: [obm-l] RE: [obm-l] Re: [obm-l] Socorro em geometria (construçã o)

2012-09-11 Por tôpico Luís Lopes




Sauda,c~oes, oi Rogerio, 

Vc tem razão. Vi isso depois de ter mandado a mensagem. 
O problema é quase igual ao que originou estas mensagens: 

===
Construir o trapézio ABCD conhecendo a soma das bases AB + CD = s, 
as diagonais AC = p e BD = q e o lado AD = a. Justifique.   
   
===
Esse foi resolvido. 

Imaginei um com a diferença das bases: 

Construir o trapézio ABCD conhecendo a diferença das bases AB - CD = s, 
as diagonais AC = p e BD = q e o lado DA = d. 

Mas este problema não é construtível com RC. 

Como não consegui imaginar uma construção, mandei-o para uma outra lista. 
E obtive a seguinte resposta: 

=
from my calculations it seems that AB=a must satisfy the cubic equation

2 a^3 - 3 s a^2 + (2 d^2 - p^2 - q^2 + s^2) a - d^2 s + q^2 s = 0, 

hence a is not always constructible.
=

[]'s 
Luís 


Date: Tue, 11 Sep 2012 11:07:13 -0300
Subject: [obm-l] Re: [obm-l] RE: [obm-l] Re: [obm-l] Socorro em geometria 
(construçã o)
From: abrlw...@gmail.com
To: obm-l@mat.puc-rio.br

Ola' Luis,
com apenas 3 elementos conhecidos ( a diferenca das bases, e as diagonais ) o 
terceiro problema esta' indeterminado.
[]'s
Rogerio Ponce



Em 8 de setembro de 2012 12:45, Luís Lopes qed_te...@hotmail.com escreveu:





Sauda,c~oes, 

 Por T passe uma paralela a AT. Com centro em A desenhe um arco de raio a. Esse
Typo. Por C  

Ou seja, TC é o transformado de BD pela translação do vetor DC. 

Outras construções: construir um trapézio ABCD, conhecendo-se as bases 

AB=a, CD=c, e 

1) os dois lados, BC=b e AD=d. 

2) as diagonais AC=p e BD=q. 

3) a diferença das bases AB - CD = d, as diagonais AC = p e BD = q. 

1) e 2) são problemas do tipo: dado o comprimento e a direção de um segmento, 

apoiá-lo em duas linhas (curvas). 

Apesar de parecido com o que foi acabado de ser construído, não consegui 
resolver o 3). Ou seja, não descobri a translação conveniente. 

Considero a disposição dos vértices como em 


http://en.wikipedia.org/wiki/Trapezoid

Luís 


 From: saldana...@pucp.edu.pe

 To: obm-l@mat.puc-rio.br
 CC: 
 Subject: [obm-l] Re: [obm-l] Socorro em  geometria (construçã o)
 Date: Sat, 8 Sep 2012 06:51:19 -0500

 
 
 
 Contrói o triângulo ACT com lados AC=p, CT=q e AT=s.
 
 Por T passe uma paralela a AT. Com centro em A desenhe um arco de raio a. Esse
 arco vai cortar à paralela (tem 2 soluções). Chame de D a ese ponto de corte.

 Por D traçe uma paralea a CT, o ponto de corte entre essa paralea e AT será o
 ponto B do trapecio ABCD procurado.
 
 Julio Saldaña
 
 
 -- Mensaje original ---
 De : obm-l@mat.puc-rio.br

 Para : obm-l@mat.puc-rio.br
 Fecha : Sat, 8 Sep 2012 03:09:59 +
 Asunto : [obm-l] Socorro em  geometria (construçã o)
 
 

 
 
 Construir o trapézio ABCD conhecendo a soma das bases AB +CD = s,as diagonais
 AC = p e BD = q e o lado AD = a.Justifique.   

 
 __

 Si desea recibir, semanalmente, el Boletín Electrónico de la PUCP, ingrese a:
 http://www.pucp.edu.pe/puntoedu/suscribete/
 

 =
 Instruções para entrar na lista, sair da lista e usar a lista em
 http://www.mat.puc-rio.br/~obmlistas/obm-l.html

 =
  


  

[obm-l] RE: [obm-l] Re: [obm-l] Socorro em geometria (construçã o)

2012-09-08 Por tôpico Luís Lopes

Sauda,c~oes, 

 Por T passe uma paralela a AT. Com centro em A desenhe um arco de raio a. Esse
Typo. Por C  

Ou seja, TC é o transformado de BD pela translação do vetor DC. 

Outras construções: construir um trapézio ABCD, conhecendo-se as bases 
AB=a, CD=c, e 

1) os dois lados, BC=b e AD=d. 

2) as diagonais AC=p e BD=q. 

3) a diferença das bases AB - CD = d, as diagonais AC = p e BD = q. 

1) e 2) são problemas do tipo: dado o comprimento e a direção de um segmento, 
apoiá-lo em duas linhas (curvas). 

Apesar de parecido com o que foi acabado de ser construído, não consegui 
resolver o 3). Ou seja, não descobri a translação conveniente. 

Considero a disposição dos vértices como em 

http://en.wikipedia.org/wiki/Trapezoid

Luís 


 From: saldana...@pucp.edu.pe
 To: obm-l@mat.puc-rio.br
 CC: 
 Subject: [obm-l] Re: [obm-l] Socorro em  geometria (construçã o)
 Date: Sat, 8 Sep 2012 06:51:19 -0500
 
 
 
 Contrói o triângulo ACT com lados AC=p, CT=q e AT=s.
 
 Por T passe uma paralela a AT. Com centro em A desenhe um arco de raio a. Esse
 arco vai cortar à paralela (tem 2 soluções). Chame de D a ese ponto de corte.
 Por D traçe uma paralea a CT, o ponto de corte entre essa paralea e AT será o
 ponto B do trapecio ABCD procurado.
 
 Julio Saldaña
 
 
 -- Mensaje original ---
 De : obm-l@mat.puc-rio.br
 Para : obm-l@mat.puc-rio.br
 Fecha : Sat, 8 Sep 2012 03:09:59 +
 Asunto : [obm-l] Socorro em  geometria (construçã o)
 
 
 
 
 Construir o trapézio ABCD conhecendo a soma das bases AB +CD = s,as diagonais
 AC = p e BD = q e o lado AD = a.Justifique.   

 
 __
 Si desea recibir, semanalmente, el Boletín Electrónico de la PUCP, ingrese a:
 http://www.pucp.edu.pe/puntoedu/suscribete/
 
 =
 Instruções para entrar na lista, sair da lista e usar a lista em
 http://www.mat.puc-rio.br/~obmlistas/obm-l.html
 =
  

[obm-l] FW: solicitação

2012-07-27 Por tôpico Luís Lopes

Sds, 
Alguém pode ajudar? Obrigado. 
Luis



Subject: solicitação
Date: Fri, 27 Jul 2012 08:14:16 -0300





Em uma escola 95% das pessoas tem 1,62 m ou mais e 8% 1,62,m ou menos. Calcule 
o % de quem tem 1,62 .Solicito uma ajuda nesta questão. 
  

  

[obm-l] FW: solicitação

2012-07-27 Por tôpico Luís Lopes

Sds, 
Reenviando. 
From: qed_texte@hotmail.comTo: ob...@mat.puc-rio.brSubject: FW: 
solicitaçãoDate: Fri, 27 Jul 2012 12:12:47 +Sds, Alguém pode ajudar? 
Obrigado. LuisSubject: solicitaçãoDate: Fri, 27 Jul 2012 08:14:16 -0300Em uma 
escola 95% das pessoas tem 1,62 m ou mais e 8% 1,62,m ou menos. Calcule o % de 
quem tem 1,62 .Solicito uma ajuda nesta questão. 
  

[obm-l] FW: PROBLEMAS..... de concurso??

2012-06-25 Por tôpico Luís Lopes

Sauda,c~oes, 
Me mandaram os problemas abaixo com o gabarito. Que tirei para ver as respostas 
justificadas de vocês, sempre melhores e mais espertas do que as minhas. 
Faço isso por 3 razões:
1) para me ajudarem; 2) para dar uma melhor resposta ao Fernando; 3) para tirar 
a lista do silêncio e moviment'a-la um pouco. 
[ ]'s Lu'is 

 
Prezado Luis,
 
Gostaria de sua ajuda para as seguintes 
questões:


1)Se 
dois trinômios do 2º grau possuem as mesmas raízes então:
a) eles são 
necessariamente iguais.
b) eles assumem necessariamente um mínimo ou um 
máximo no mesmo ponto.
c) eles diferem por uma constante.
d) suas 
concavidades são de mesmo sentido.
e) nenhuma das anteriores.
R. letra  a letra d é f'acil de ser eliminada. hum a letra a também
 
2)Dados três pontos no plano cartesiano, não colineares e com abscissas 
distintas duas a duas, o número de funções quadráticas que podem ser 
encontradas 
de maneira que esses pontos pertençam aos seus gráficos é:
a) 
0  b) 
1  c) 
2  d) 
3
R.letra .? 
  

[obm-l] conjectura sobre colinearidade

2012-05-29 Por tôpico Luís Lopes

Sauda,c~oes, 

Retomo uma (muito) velha mensagem. 

Continuo ao final das mensagens (nada como um 
bom sistema de arquivamento). 

O Claudio Buffara ainda acessa a lista? 

-Mensagem Original-
De: Claudio Buffara claudio.buff...@terra.com.br
Para: obm-l@mat.puc-rio.br
Enviada em: sábado, 6 de março de 2004 01:42
Assunto: Re: [obm-l] conjectura sobre colinearidade


  Sauda,c~oes,
 
  Seja dado o triangulo AP_0Q_0 .
 
  Em AP_0 e AQ_0 marcamos P_0Q_i,
  e Q_0P_i  tais que P_0Q_i = Q_0P_i = m_i,
  i = 1,2,  e m_i  m_{i+1} (todos diferentes
  entre si). Unimos P_0P_i  e  Q_0Q_i,
  obtendo a interseção R_i.
 
  Conjectura: os R_i são colineares.
 
  Como provar? Qual a teoria que suporta
  tal resultado? Teorema de Desargue?
 
  Se a conjectura vira um teorema, temos
  uma solução para os problemas
  A,a+b,a-c  e  A,a-b,a-c.
Typo: A,a+b,a+c  e  A,a-b,a-c.

 
  []'s
  Luís
 
 
 Oi, Luis:

 A conjectura eh verdadeira. Veja a seguir...

 Considere o triangulo APQ e vetores unitarios u e v 
 nas direcoes PA e QA, respectivamente. Se |PA| = b e 
 |QA| = c, entao teremos que o vetor PQ serah bu - cv.

 Sejam Q' sobre AP e P' sobre AQ tais que |PQ'| = |QP'| = m.
 Entao, PQ' = mu  e  QP' = mv.

 PP' = bu - cv + mv = bu + (m-c)v
 QQ' = mu - (bu - cv) = (m-b)u + cv

 Interseccao de PP' e QQ' == existem x e y reais tais que:
 R = PQ + x*QQ' = y*PP' ==
 bu - cv + x*((m-b)u + cv) = y*(bu + (m-c)v) ==
 (b + (m-b)x - by)u + (-c + cx - (m-c)y)v = 0

 Como o triangulo APQ eh nao degenerado, u e v sao L.I.
 Assim:
 (m-b)x - by = -b
 cx - (m-c)y = c

 Resolvendo este sistema, obtemos:
 x = b/(b+c-m)  e  y = c/(b+c-m)

 O ponto de interseccao serah:
 R = y*PP' = c(bu + (m-c)v)/(b+c-m)

 dR/dm = bc(u + v)/(b+c-m) = multiplo de um 
 vetor constante (u + v) == ao se variar m, 
 R percorre uma linha reta == CQD

 Um abraco,
 Claudio.


Tudo muito bem. Hoje sei (ver a mensagem de 08/03/04) 
http://www.mat.puc-rio.br/~nicolau/olimp/obm-l.200403/msg00294.html
que a conjectura é verdadeira usando feixes anarmônicos, 
razões anarmônicas (cross-ratios), raios homólogos 
e feixes perspectivos. 
Tudo isso é geometria projetiva com uma abordagem cearense. 

A reta suporte dos pontos R (interseção de raios homólogos) 
é o eixo da perspectiva. 
Ou da homologia. E o vértice A, o centro da homologia. 

Acompanhei a demonstração do Buffara e acho que tá 
tudo ok. Ou quase. 

Empaquei aqui. 

 O ponto de interseccao serah:
 R = y*PP' = c(bu + (m-c)v)/(b+c-m)
Ok. Então R=f(m), como esperado. 

 dR/dm = bc(u + v)/(b+c-m) = multiplo de um 
 vetor constante (u + v) == ao se variar m, 
 R percorre uma linha reta == CQD
Não seria dR/dm = bc(u + v)/(b+c-m)^2 ?? 

E como R percorre uma linha reta? 

dR/dm = k(u+v) múltiplo de um vetor constante 
(u+v). Onde k=bc/(b+c-m)^2. 

Para R percorrer uma reta, k não teria que ser 
independente de m também??? 

Gostaria de comentários, correção, confirmação 
sobre o final da mensagem do Buffara. 

Obrigado. 

Luís 

  

[obm-l] questoes EPCAR

2012-05-22 Por tôpico Luís Lopes




Sauda,c~oes, 

Alguém pode resolver? Obrigado. 

Abs, 
Luís 



01) EPCAR - 2007 A dá a B tantos reais quantos B possui e A dá a C 
tantos reais quantos C possui. 
Depois, B dá a A e a C tantos reais 
quantos cada um possui e C, finalmente, faz a mesma coisa. 
Se no final, 
terminam todos com 16 reais e sabendo que C começou com 50% de B mais um
 real, 
então A começou com 

a) 24 reais  
 c) 28 reais b) 26 reais
   d) 30 reais 


02) EPCAR - 2007 
Uma loja colocou um CD à venda por R$ 28,00 a unidade. 
Como não atraiu 
muitos compradores, resolveu baixar o preço para um 
número inteiro de 
reais. Com isso, vendeu o restante do estoque 
que não era superior a 50 
unidades, por R$ 377,00. Com base nisso, 
o número n de unidades do CD 
restante no estoque é um número 
cuja soma dos algarismos vale 
 
a)
 6  c) 11 b) 9  
d) 15   




  

RE: [obm-l] Soma

2012-04-22 Por tôpico Luís Lopes

Sauda,c~oes, 





Não seria 





1 + 2*2^1 + 3*2^2 + 4*2^3 + ... + n*2^(n-1) ? 





O termo geral a_k pode ser escrito como a_k = (a_1 + (k-1)r)q^{k-1} 





com a_1=r=1 e q=2. 

Temos então uma progressão aritmético-geométrica cuja fórmula fechada 
para a soma S_n = \sum_{k=1}^n a_k  é

S_n = \frac{a_1(q^n - 1)}{q-1} + \frac{qrA}{(q-1)^2} com 

A = 1 - nq^{n-1} + (n-1)q^n 

Substituindo a_1=r=1 e q=2, vem:

S_n = (n-1)2^n + 1

Trato deste assunto, bem como de material que complementa o que o 
Nehab mandou recentemente sobre Médias no Manual de Progressões. 

Abs, 
Luís 


 Date: Sun, 22 Apr 2012 08:08:53 -0300
 From: smo...@terra.com.br
 To: obm-l@mat.puc-rio.br
 Subject: Re: [obm-l] Soma
 
 Ops... cometi o velho erro de trocar o sinal. resposta final deve ser 
 (n-1).(2^n) - 1


 =
 Instruções para entrar na lista, sair da lista e usar a lista em
 http://www.mat.puc-rio.br/~obmlistas/obm-l.html
 =
  

[obm-l] RE: [obm-l] Re: [obm-l] RE: [obm-l] RE: [obm-l] Re: [obm-l] RE: [obm-l] RE: [obm-l] Provar que é irracional...

2012-04-03 Por tôpico Luís Lopes

Sauda,c~oes, 

Defina a sequencia f_0=0 e f_(n+1)=raiz(2+f_n) para n=0,1,2,...

A solução desta recorrência é f_n=2cos(pi/2^{n+1}). Logo, 
cos(pi/4,8,16) são irracionais. E lim f_n=2. 

Abs, 
Luís 


Date: Sun, 1 Apr 2012 16:02:54 -0300
Subject: [obm-l] Re: [obm-l] RE: [obm-l] RE: [obm-l] Re: [obm-l] RE: [obm-l] 
RE: [obm-l] Provar que é irracional...
From: ralp...@gmail.com
To: obm-l@mat.puc-rio.br

Bom, do jeito que eu escrevi seria f_0=0. Entao voce tinha razao quando disse 
que eu estava errado. Eu acho. :) Abraco,  Ralph


2012/4/1 marcone augusto araújo borges marconeborge...@hotmail.com





Esqueça.claro,f_0=2.Obrigado.
 




From: marconeborge...@hotmail.com
To: obm-l@mat.puc-rio.br
Subject: [obm-l] RE: [obm-l] Re: [obm-l] RE: [obm-l] RE: [obm-l] Provar que é 
irracional...

Date: Sat, 31 Mar 2012 01:01:42 +





Era sim. 
f_0=0,não?
 




Date: Sun, 25 Mar 2012 17:59:28 -0300
Subject: [obm-l] Re: [obm-l] RE: [obm-l] RE: [obm-l] Provar que é irracional...
From: ralp...@gmail.com
To: obm-l@mat.puc-rio.br



Defina a sequencia f_0=2 e f_(n+1)=raiz(2+f_n) para n=0,1,2,...
 
Note que f_1=raiz(2) eh irracional (bom, espero que isto tenha sido demonstrado 
anteriormente).
 
Agora, note que se f_(n+1) fosse RACIONAL, entao f_n=(f_(n+1))^2-2 tambem seria 
RACIONAL. Ou seja, se f_n eh IRRACIONAL, entao f_(n+1) eh IRRACIONAL.
 
Assim, como f_1 eh irracional, por inducao, todos os f_n sao irracionais 
(n=1,2,3,...).
 
Era isso?
 
Abraco,
  Ralph


2012/3/24 marcone augusto araújo borges marconeborge...@hotmail.com



Obrigado.Eu vi essa questão numa lista de indução.
Vejo uma idéia de indução ai,mas,se não for abusar da sua boa vontade,como 
seria uma solução com um
procedimento mais explicito de indução? 

  
 





From: joao_maldona...@hotmail.com
To: obm-l@mat.puc-rio.br
Subject: [obm-l] RE: [obm-l] Provar que é irracional...

Date: Sat, 24 Mar 2012 19:34:57 -0300




Bom, sendo f(x) = raiz(2 +raiz(2 + raiz(2+...), x vezes, é óbvio que f(x+1)  
f(x), Logo o valor máximo é f(infinito), mas se x tende ao infinito, temos que 
f(x) = raiz(2 + f(x)), que elevando ao quadrado temos f(x) = 2, logo para 
qualuqer x diferente do infinito (que é o caso), f(x)  2, além disso f(x)  0 
e f(x) = f(1) = raiz(2) =~ 1.4 


Elevando ao quadrado desse modo:
f(x) = raiz(2 +raiz(2 + raiz(2+...)  - f(x)² - 2 = f(x-1) - (f(x)²-2)²-2 = 
f(x-2), repetindo isso x vezes temos -
 ((f(x)²-2)²-2)²-2...²-2)=0, que expandindo tem coeficiente lider 1 e 
termo independendo -2, logo pelo teorema das raízes racionais, se f(x) é 
racional, é -2, -1, 1, ou 2, absurdo, logo f(x) é irracional.


[]'s
João





From: marconeborge...@hotmail.com
To: obm-l@mat.puc-rio.br
Subject: [obm-l] Provar que é irracional...

Date: Sat, 24 Mar 2012 21:56:30 +


Como provar q raiz(2+raiz(2+raiz(2+...raiz(2)),generalizando para n raizes,é 
irracional? 
  


  

RE: [obm-l] Conta Matematica

2012-03-01 Por tôpico Luís Lopes

Sauda,c~oes, oi Bernardo, Gmerencio, 

Enfim, concordo com o Bernardo e não vejo nada de muito polêmico na questão.
Isso. No papel todos deveriam encontrar 9 sem nenhuma dúvida. 

Não seria a primeira vez que falhas envolvendo calculadoras Casio 
acontecem:Obrigado pelo link (histórico). Então a (minha) pergunta é: 

O que levou a calculadora ter tal comportamento? 

Que fica difícil de ser respondida fora do contexto. 

Abs, 
Luís 


Date: Thu, 1 Mar 2012 10:05:50 -0300
Subject: Re: [obm-l] Conta Matematica
From: gmerencio.san...@gmail.com
To: obm-l@mat.puc-rio.br

Não seria a primeira vez que falhas envolvendo calculadoras Casio acontecem:
http://www.washingtonpost.com/wp-srv/aponline/2226/aponline161058_000.htm 
http://media.newtella.de/get/media/ObDDCXkwk8L6gZ9blf8t/525

Enfim, concordo com o Bernardo e não vejo nada de muito polêmico na questão. O 
único modo de obter 1 como resultado é calculando 6/(2(1 + 2)), porque é 
avaliado o que está dentro dos parênteses e depois realizada a divisão. 
Provavelmente o engano é devido à intuição: é comum escrever 6/2(1 + 2), quando 
na verdade queremos dizer 6/(2(1 + 2)). Mas máquinas são (ou deveriam ser) 
exatas, com instruções claras e precisas.

2012/3/1 Bernardo Freitas Paulo da Costa bernardo...@gmail.com

2012/3/1 Luís Lopes qed_te...@hotmail.com:

 Sauda,c~oes,



 Num forum familiar o que segue abaixo está pegando fogo.



 E estou sendo muito cobrado a dar uma resposta definitiva. :)



 Que não possuo.



 O debate está aberto aqui mas gostaria de conhecer o histórico

 da questão. Pois se não for armação acho que devem ter

 rolado algumas cabeças no fabricante da calculadora.

Bom, eu não conheço o histórico, mas a minha opinião é que as regras

de preferência de operadores são um tanto explícitas em matemática:

- parênteses, ou qualquer outra forma de grupos com início e fim

(colchetes, chaves, barras, ...)

- exponenciação

- multiplicação e divisão

- adição e subtração



Quando há duas operações do mesmo nível, resolve-se da esquerda para a

direita. Assim, a + b - c = (a + b) - c, a - b + c = (a - b) + c, a +

b - c - d + e + f - g = ( ... (a + b) - c) - d ) + e ) + f ) - g



Mais ainda, o sinal de multiplicação pode ser omitido quando for

claro. Assim, 45(a+b) = 45 * (a + b).



Portanto, 6/2(1+2) = 6/2*(1+2) = 6/2*3 = (6/2)*3 = 9.



É claro que você poderia tentar justificar o outro resultado dizendo

que a multiplicação implícita tem preferência sobre os demais

operadores mas não sobre os grupos. Isso não faz nenhum sentido para

mim, porque inclui um novo tipo de elemento na sua árvore sintática

(se é que a calculadora implementa uma árvore sintática para analisar

as expressões, da mesma forma que um compilador faz na hora de

construir as instruções), além de criar uma regra a mais para decorar.



 Abs,

 Luís



 http://9gag.com/gag/2957368





--

Bernardo Freitas Paulo da Costa



=

Instruções para entrar na lista, sair da lista e usar a lista em

http://www.mat.puc-rio.br/~obmlistas/obm-l.html

=


  

RE: [obm-l] fatorar polinomio

2012-01-25 Por tôpico Luís Lopes

Sauda,c~oes, 

Obrigado João e Bernardo pelo interesse. 

Vou começar pelo começo. 

Quero resolver 

4b^4 + 4b^3 + 2b^2 - 4b + 1 = 0. 

Seguindo a receita, faço b = y/4 e caio em 

y^4 + 4y^3 + 8y^2 - 64y + 64 = 0. 

Faço y = x - 1 e elimino o termo em y^3. 

Vem 

x^4 + px^2 + qx + r = 0 (*)
com p=2, q=-72, r=133. 

A cúbica resolvente de (*) é 

u^3 - 2u^2 - 532u - 4120 = 0 

A cúbica possui (como esperado) uma raiz racional. 
u_1=-10. As outras são u_2=6-8\sqrt7 e u=6+8\sqrt7. 

Pegando u_1 enrola as contas. Aí mandei a primeira mensagem. 
Então vi que deveria pegar u . 

As raízes de (*) saem da equação 

(x^2 + u/2)^2 = (Ux + V)^2 

com U=\sqrt{4 + 8\sqrt7} e V= \sqrt{24\sqrt7 - 12}. 

Desfazendo as duas transformações, obtemos 
a fatoração do polinômio original, na forma 

(2b^2 + Ab + B)(2b^2 + Cb + D). 

Vem: 

A = 1 - \sqrt{1 + 2\sqrt7} 

B = \frac{P}{4}

C = 1 + \sqrt{1 + 2\sqrt7} 


D = \frac{Q}{4}


com 

P = 2(1 + \sqrt7) - (\sqrt{1 + 2\sqrt7} + \sqrt{6\sqrt7 - 3})

Q = 2(1 + \sqrt7) + (\sqrt{1 + 2\sqrt7} + \sqrt{6\sqrt7 - 3})

Voltarei à origem desse problema numa outra 
mensagem. 

Abs, 
Luís 





From: joao_maldona...@hotmail.com
To: obm-l@mat.puc-rio.br
Subject: RE: [obm-l] fatorar polinomio
Date: Mon, 23 Jan 2012 19:43:37 -0200








http://www.wolframalpha.com/input/?i=A%2BC%3D4%2C++D+%2BB+%2B+++A+C++%3D+8%2C+A+D+%2B+B+C+%3D+-64%2C++B+D+%3D+64
 

Em Real Soluctions
Tem a opção Exact Form, é só clicar lá  que aparece a  forma exata
[]'sJoão

From: qed_te...@hotmail.com
To: obm-l@mat.puc-rio.br
Subject: [obm-l] fatorar polinomio
Date: Mon, 23 Jan 2012 20:55:02 +







Sauda,c~oes, 

Gostaria de fatorar o polinômio 

x^4 + 4x^3 + 8x^2 - 64x + 64 na forma 

(x^2 + Ax + B)(x^2 + Cx + D). 

Usando WolframAlpha o máximo que consegui foi 

A=-3.0165735805
B=2.390218772169901
C=7.0165735805
D=26.775791716886353

Gostaria dos valores exatos (forma fechada) 
para A, B, C, D. 

Antes de usar um programa algébrico tentei resolver 
a quártica. A cúbica resolvente tem L=-10 como raiz 
e voltando pra quártica, temos: 

(x^2 - 5)^2 = -12x^2 + 72x - 108 

(x^2 - 5)^2 =  -(12x^2 - 72x + 108) 


(x^2 - 5)^2 =  i^2 (2\sqrt{3} x - 6\sqrt{3})^2 

Pensei que resolvendo (x^2 - 5) =  i (2\sqrt{3} x - 6\sqrt{3}) e 

(x^2 - 5) =  -i (2\sqrt{3} x - 6\sqrt{3}) 

iria encontrar as raízes da quártica e daí a sua fatoração. 

Mas as contas complexas complexas :) não me 
permitiram chegar a lugar nenhum. 

Uma resposta com um programa já seria bom mas gostaria 
de ter o desenvolvimento completo dos cálculos. 

Obrigado. 

Abs, 
Luís 



  

[obm-l] raizes construtiveis

2012-01-25 Por tôpico Luís Lopes

Sauda,c~oes, 

Como dito na mensagem anterior, voltemos 
ao estudo de uma quártica. 

e_c = bissetriz externa do vértice C. 
Problema: Construir o triângulo ABC dados 
a,h_b,e_c ou a,h,e. Esta construção 
é imediata e o problema tem (pode ter) 
4 soluções. 

Algebricamente, o lado b é raiz da quártica 

(4a^2h^2 + e^4 - 4a^2e^2)b^4 + (8a^3e^2 - 4ae^4)b^3 
+ (6a^2e^4 - 4a^4e^2)b^2 - 4a^3e^4 b + a^4e^4 = f(b). 

Assim, se a,h,e são números construtíveis, então as 
4 raízes de f(b) também são construtíveis e a cúbica 
resolvente tem uma (pelo menos) raiz racional. 

Os casos complicados aparecem quando os dados não 
permitem a construção dos triângulos, como ha. 

Exemplos: 

1) a=e=1, h=0 f(b) = 0 torna-se 

3b^4 - 4b^3 - 2b^2 + 4b - 1 = 0 

e f(b) = (3b-1)(b+1)(b-1)^2 

Raízes reais que não formam um triângulo. 

2) a=e=1, h=(\sqrt7)/2 f(b) = 0 torna-se 

4b^4 + 4b^3 + 2b^2 - 4b + 1 = 0

Vimos na mensagem anterior que a cúbica 
resolvente tem uma raiz racional e as 4 raízes 
da quártica são complexas. 

Estes e outros exemplos me levaram a fazer 
a seguinte conjectura: 

Se a,h,e são números construtíveis, então 

As raízes de f(b) são sempre construtíveis. 
Ou a cúbica resolvente de f(b) possui sempre 
uma - pelo menos - raiz racional. 

Seria possível provar ou refutar esta afirmação? 

Dar condições para que ela seja verdadeira? 

Abs, 
Luís 

  

[obm-l] fatorar polinomio

2012-01-23 Por tôpico Luís Lopes

Sauda,c~oes, 

Gostaria de fatorar o polinômio 

x^4 + 4x^3 + 8x^2 - 64x + 64 na forma 

(x^2 + Ax + B)(x^2 + Cx + D). 

Usando WolframAlpha o máximo que consegui foi 

A=-3.0165735805
B=2.390218772169901
C=7.0165735805
D=26.775791716886353

Gostaria dos valores exatos (forma fechada) 
para A, B, C, D. 

Antes de usar um programa algébrico tentei resolver 
a quártica. A cúbica resolvente tem L=-10 como raiz 
e voltando pra quártica, temos: 

(x^2 - 5)^2 = -12x^2 + 72x - 108 

(x^2 - 5)^2 =  -(12x^2 - 72x + 108) 


(x^2 - 5)^2 =  i^2 (2\sqrt{3} x - 6\sqrt{3})^2 

Pensei que resolvendo (x^2 - 5) =  i (2\sqrt{3} x - 6\sqrt{3}) e 

(x^2 - 5) =  -i (2\sqrt{3} x - 6\sqrt{3}) 

iria encontrar as raízes da quártica e daí a sua fatoração. 

Mas as contas complexas complexas :) não me 
permitiram chegar a lugar nenhum. 

Uma resposta com um programa já seria bom mas gostaria 
de ter o desenvolvimento completo dos cálculos. 

Obrigado. 

Abs, 
Luís 


  

[obm-l] FW: PROBLEMA DE LÓGICA

2011-12-09 Por tôpico Luís Lopes

Sauda,c~oes, 

O Rafael me pediu ajuda para tirá-lo da enrascada abaixo. 

A resposta está certa? Qual a maneira rápida/elegante de 
se resolver tais problemas? 

Abraços, 
Luís 







=



Meu caro Luís,
 
Gostaria de sua ajuda para  a seguinte 
questão:

 
EXERCÍCIO Nº 1: Quatro amigos vão ao rodeio em 
Barretos e um
deles entra sem pagar ingresso. Um fiscal do evento 
quer saber
quem foi o penetra: Eu não fui, diz o Augusto. Foi o 
Chicão, diz o
Leonardo. Foi o Leonardo, diz o Rafael. Rafael não 
tem razão, diz
o Chicão. Sabendo que só um deles mentiu. Quem não 
pagou a 
entrada?



a) Leonardo
b) Chicão
c) Rafael
d) Augusto
e) n.d.a
Resposta: C.

  

[obm-l] RE: [obm-l] Solução da série \sum_{i=1}^n 2^{n-i}i^2

2011-09-13 Por tôpico Luís Lopes

Sauda,c~oes, oi André, 

Comece calculando a série \sum_{i=1}^n i x^i. 

E depois \sum_{i=1}^n i^2 x^i. 

Para os detalhes, ver os exercícios 47 e 54 do Manual 
de Seq. e Series Vol I em http://www.escolademestres.com

Abs, 
Luís 



From: andrerc...@gmail.com
Date: Mon, 12 Sep 2011 20:06:35 -0300
Subject: [obm-l] Solução da série \sum_{i=1}^n 2^{n-i}i^2
To: obm-l@mat.puc-rio.br

Pessoal,Alguém pode me dar alguma dica como se consegue obter a fórmula fecha 
da série \sum_{i=1}^n 2^{n-i}i^2O Wolfram Alpha indica que a fórmula fechada da 
mesma é 3 * 2^{n+1} - n^2 - 4n - 6:



http://www.wolframalpha.com/input/?i=%5Csum_{i%3D1}^n+2^{n-i}i^2
Porém, até então não obtive êxito.



Abraço,__
André




  

[obm-l] 2^70 + 3^70 eh divisivel por 13

2011-08-10 Por tôpico Luís Lopes

Sauda,c~oes, 

Alguém poderia resolver? 

Solicitaria a voce uma solução para a questão : 
demonstre que 270 + 370 é divisível por 13. 

  

  
[]'s 
Luis 

  

[obm-l] RE: [obm-l] Re: [obm-l] Re: [obm-l] dúvida sobre séries

2011-06-07 Por tôpico Luís Lopes

Sauda,c~oes, 

Legal este critério, parece ter sido criado para a série harm. 

E a esse respeito, o autor da pergunta poderia ler também sobre 
a constante de Euler. 

[]'s 
Luís 

 Date: Mon, 6 Jun 2011 23:50:37 -0300
 Subject: [obm-l] Re: [obm-l] Re: [obm-l] dúvida sobre séries
 From: rodrigo.uff.m...@gmail.com
 To: obm-l@mat.puc-rio.br
 
 Olá!
 
 Uma outra maneira  ( além da que os colegas enviaram antes), para
 mostrar que a série  não converge, tem um critério de convergência que
 acho legal, Critério de condensação de Cauchy:
 
 
  Se x_k é uma sequência decrescente de termos positivos ( como é o caso de 
 1/k )
 
 
 então a série [ SOMA de x_k]  converge , se e somente se , a série [
 SOMA de 2^k  x_(2^k) ]  converge.
 
 Aplicando isso para a série do email
 
 temos com a_k= 1/k
 
 
 [ SOMA de 2^k  x_(2^k) ]  =  [ SOMA de 2^k  ,  1/ (2^k)   ] =  [ SOMA 1 ]
 
 que diverge, pois somando de 1 até n resulta em n, com n indo pro
 infinito , diverge  : ) Pode não ajudar muito, mas acho esse critério
 legal
 
 abraço
 
 =
 Instruções para entrar na lista, sair da lista e usar a lista em
 http://www.mat.puc-rio.br/~obmlistas/obm-l.html
 =
  

[obm-l] Geometria Cone Sul

2011-05-12 Por tôpico Luís Lopes

Sauda,c~oes, 

Fonte: Treinamento Cone Sul Volume 2. 

Problema 26 p. 135 

H_b , H_c pés das alturas de B e C. 
H ortocentro 
M_a médio de BC 
Gamma Circuncírculo de ABC 
phi Circuncírculo de AH_bH_c 
S segunda interseção de phi com Gamma 

Mostre que S, H, M_a são colineares. 

Como fazer? Com geometria sintética de preferência. 

[]'s 
Luís 

  

RE: [obm-l] Fwd: Identidade de Euler

2011-05-01 Por tôpico Luís Lopes

Sauda,c~oes, 

Este é o exercício 14 do Manual de Seq. e Séries Volume 2. 

A página http://www.escolademestres.com/ 
contém uma amostra dele. 

[]'s 
Luís 


Date: Thu, 28 Apr 2011 17:40:10 -0300
Subject: Re: [obm-l] Fwd: Identidade de Euler
From: victorcar...@globo.com
To: obm-l@mat.puc-rio.br

Oi  Mestre  Nehab ,
Gostei da sugestão e mais ainda  das n pessoas que moram em Nilópolis ( minha  
terrinha).
 
Abraços 
 
Carlos  Victor


Em 28 de abril de 2011 17:21, Carlos Nehab ne...@infolink.com.br escreveu:

Oi, Fábio,

Não resisti:

Resolva os seguinte problema de duas maneiras (uma técnica básica e útil para 
resolver identidades deste tipo).

De quantas maneira posso formar comissões de p pessoas, a partir de um total de 
m + n pessoas, sendo m o total de pessoas que moram no Maracanã e n as pessoas 
que moram em Nilópolis?

Abraços,
Nehab

Em 28/4/2011 13:24, fabio henrique teixeira de souza escreveu: 




-- Mensagem encaminhada --
De: fabio henrique teixeira de souzafabiodja...@ig.com.br

Data: 28 de abril de 2011 08:52
Assunto: Identidade de Euler
Para: obm-l@mat.puc-rio.br


Pessoal, estou batendo cabeça e não consigo demonstrar que

C(m,0).C(n,p) + C(m,1).C(n,p-1) + C(m,2).C(n,p-2) + ... + C(m,p).C(n,0) =
C(m+n,p)

Alguém pode me dar uma dica?


=

Instruções para entrar na lista, sair da lista e usar a lista em
http://www.mat.puc-rio.br/~obmlistas/obm-l.html
=


  

RE: [obm-l] Fwd: Identidade de Euler

2011-04-29 Por tôpico Luís Lopes

Sauda,c~oes, 





Este é o exercício 14 do Manual de Seq. e Séries Volume 2. 
A resolução lá apresentada é outra: identidade de polinômios 
(uma outra técnica básica e útil para resolver identidades deste tipo). 





A página http://www.escolademestres.com/ 


contém uma amostra dele. 





[]'s 


Luís 



Date: Thu, 28 Apr 2011 17:40:10 -0300
Subject: Re: [obm-l] Fwd: Identidade de Euler
From: victorcar...@globo.com
To: obm-l@mat.puc-rio.br

Oi  Mestre  Nehab ,
Gostei da sugestão e mais ainda  das n pessoas que moram em Nilópolis ( minha  
terrinha).
 
Abraços 
 
Carlos  Victor


Em 28 de abril de 2011 17:21, Carlos Nehab ne...@infolink.com.br escreveu:

Oi, Fábio,

Não resisti:

Resolva os seguinte problema de duas maneiras (uma técnica básica e útil para 
resolver identidades deste tipo).

De quantas maneira posso formar comissões de p pessoas, a partir de um total de 
m + n pessoas, sendo m o total de pessoas que moram no Maracanã e n as pessoas 
que moram em Nilópolis?

Abraços,
Nehab

Em 28/4/2011 13:24, fabio henrique teixeira de souza escreveu: 




-- Mensagem encaminhada --
De: fabio henrique teixeira de souzafabiodja...@ig.com.br

Data: 28 de abril de 2011 08:52
Assunto: Identidade de Euler
Para: obm-l@mat.puc-rio.br


Pessoal, estou batendo cabeça e não consigo demonstrar que

C(m,0).C(n,p) + C(m,1).C(n,p-1) + C(m,2).C(n,p-2) + ... + C(m,p).C(n,0) =
C(m+n,p)

Alguém pode me dar uma dica?


=

Instruções para entrar na lista, sair da lista e usar a lista em
http://www.mat.puc-rio.br/~obmlistas/obm-l.html
=


  

RE: [obm-l] [obm-l] Geometria Analítica

2011-02-25 Por tôpico Luís Lopes

Sauda,c~oes, 

Esse lugar geométrico me lembrou de outro. Seja construir 
o triângulo ABC dados (A,r,m_a). 

Podemos construir o vértice A, o incentro I e o incírculo W. 
Construindo o ponto M_a (médio de BC) o triângulo fica 
determinado pela tangente por M_a a W (reta do lado a). 

O ponto M_a está em dois lugares: 

1) círculo U=(A,m_a) 

2) hipérbole H

Quem sabe apelando-se pro Geogebra dá pra determinar H? 
Alguém sabe fazer isso ? 

Aí fica o problema de construir com régua e compasso a interseção 
U,H. Neste caso em particular isto é possível. 

[]'s 
Luís 


From: thiago_...@hotmail.com
To: obm-l@mat.puc-rio.br
Subject: RE: [obm-l] [obm-l] Geometria Analítica
Date: Thu, 24 Feb 2011 22:57:16 -0300








Bom, eu tentei resolver. Eu não consegui, mas cheguei numa solução apelando pro 
geogebra. Se voce quiser saber, só pra  verificar quando acabar, ou sei lá, 
selecione tudo que está dentro do parentesis:
(A equação que determina o lugar geométrico de C(x,y) é a da hiperbole 3x² - y² 
- 12x + 9 = 0ou   (x - 2)² - y²/3 = 1)
Eu vou tentar mais, quem sabe não mando aqui depois uma resolução!Thiago

Date: Thu, 24 Feb 2011 15:15:59 -0300
Subject: [obm-l] [obm-l] Geometria Analítica
From: cortes...@gmail.com
To: obm-l@mat.puc-rio.br

Os extremos da base de um triângulo são A(0,0) e B(3,0). Determinar a 
equação do lugar geométrico do vértice oposto C se este se move de 
maneira que o ângulo da base CAB é sempre igual a duas vezes o ângulo da
 base CBA.

[obm-l] um prob. de teoria dos numeros e outro de otimizacao

2011-02-24 Por tôpico Luís Lopes

Sauda,c~oes, 

Acabo de receber as questões abaixo. Não conheço tais assuntos. 
Alguma sugestão? 

[]'s 
Luís 

1) Chamo produto fatorial o produto de m=2 termos consecutivos. 
de uma progressão aritmética de razão r /= 0, cujo primeiro termo 
seja a /=0, todos números naturais: indico com F(a,r,m). Assim, por 
exemplo, F(1,1,m)=m! . Não é difícil demonstrar, tomando por base 
um histórico teorema sobre a existência de números primos em 
intervalos fixos, que m! não é potência de de qualquer número natural. 
Há demonstração análoga para o caso geral F(a,r,m)? 

2) Um polígono articulado plano é um polígono plano cujos lados têm 
comprimento dado e cujos ângulos internos são variáveis. 
Fornecidos os lados de um tal polígono, é fácil determinar, a priori, 
sua área máxima, e se sua área se anula. Se, contudo, esta não se 
anula, como determinar a área mínima desse polígono articulado? 

  

RE: [obm-l] sequencia

2011-02-18 Por tôpico Luís Lopes

Sauda,c~oes, 

Este é o exercício 61 no Manual de Progressões. 
Sugestão: considere (b_n) tal que b_n=1/a_n. 

Assim b_n=(n^2 - n + 2)/2. 

E aquele outro 1 + 11 + 111 +  + 1 
é o exercício 82. 

[]'s 
Luís 


From: marconeborge...@hotmail.com
To: obm-l@mat.puc-rio.br
Subject: [obm-l] sequencia
Date: Wed, 16 Feb 2011 12:45:44 +








Determinar a1993 para a sequencia definida por a0=1 e  a(n+1)=an/(1+nan),para 
todo n natural.Desde ja agradeço.   

[obm-l] RE: [obm-l] Fórmula fechada para somatório

2011-02-16 Por tôpico Luís Lopes

Sauda, c~oes, 

Oi Henrique, 

n(n+2) = n^2 + 2n 

A soma de 2n é fácil. E a de n^2 é bem conhecida. 

De qualquer jeito este é o problema 20 no 
Manual de Seq. e Séries 1. 

O Manual de Progressões também resolve tais somas. 

Amostras em 

www.escolademestres.com/qedtexte 

[]'s 
Luís 


 Date: Wed, 16 Feb 2011 16:16:13 -0200
 Subject: [obm-l] Fórmula fechada para somatório
 From: henrique.re...@gmail.com
 To: obm-l@mat.puc-rio.br
 
 Como pode ser demonstrada a seguinte igualdade?
 
 1.3 + 2.4 + 3.5 + ... + n(n+2) = n(n+1)(2n+7)/6
 
 -- 
 Henrique
 =
 Instruções para entrar na lista, sair da lista e usar a lista em
 http://www.mat.puc-rio.br/~obmlistas/obm-l.html
 =
  

RE: [obm-l] Mais uma soma

2011-02-16 Por tôpico Luís Lopes

Sauda,c~oes, 

Novamente, o Manual de Seq. e Séries Vol 1 
mostra como calcular tais séries. 

São os exercícios 29 e 97. 

Amostras em 

www.escolademestres.com/qedtexte 

[]'s 
Luís 


 Date: Wed, 16 Feb 2011 10:38:31 -0300
 Subject: Re: [obm-l] Mais uma soma
 From: gabrieldala...@gmail.com
 To: obm-l@mat.puc-rio.br
 
 Pode-se aplicar soma telescópica, como 3/n(n+3) = 1/n - 1/(n+3)
 S = 1/1*4 + 1/4*7  + ...  + 1/196*199 = 
 (1/3)*(1/1-1/4+1/4-1/7+...+1/196-1/199)
 S = 1/3*(1-1/199) = 66/199
 
 Gabriel Dalalio
 
 Em 16 de fevereiro de 2011 09:58, marcone augusto araújo borges
 marconeborge...@hotmail.com escreveu:
  Determinar o valor da soma 1/1*4 + 1/4*7  + ...  + 1/196*199.
  Eu saberia calcular se fosse: 1*4 + 4*7 + ...196*199
  Tenho a resposta: 66/199
  Obrigado pela atençao.
 
 
 =
 Instruções para entrar na lista, sair da lista e usar a lista em
 http://www.mat.puc-rio.br/~obmlistas/obm-l.html
 =
  

[obm-l] RE: [obm-l] RE: [obm-l] Re: [obm-l] Convergência

2011-02-09 Por tôpico Luís Lopes

Sauda,c~oes, 

Com mais tempo resolvi o sistema em diferenças e expressei 
a_n numa forma fechada: Se U=\sqrt{2}, então

a_n = \frac{U(1+U)^n + U(1-U)^n}{(1+U)^n - (1-U)^n}

E lim a_n = U. 

Esta é a solução bruta. A(s) do Bernardo é a elegante. 

[]'s 
Luís 


From: qed_te...@hotmail.com
To: obm-l@mat.puc-rio.br
Subject: [obm-l] RE: [obm-l] Re: [obm-l] Convergência
Date: Tue, 8 Feb 2011 18:24:40 +








Sauda,c~oes, 

Obrigado Bernardo por tal solução. Devemos reconhecer 
e apreciar a colaboração (muitas) de membros como você. 

Eu procederia da seguinte maneira: seja o sistema 

x_{n+1} = x_n + y_n
y_{n+1} = 2*x_n + y_n

Resolvendo o sistema acima (alguém sabe como fazer isso?) 
obtemos x_n e y_n e daí a_n =  y_n/x_n. 

Então tendo a_n usamos um daqueles testes manjados para 
testar a convergência de séries (positivas). Será que daria certo? 

[]'s 
Luís 




 Date: Tue, 8 Feb 2011 16:54:04 +0100
 Subject: [obm-l] Re: [obm-l] Convergência
 From: bernardo...@gmail.com
 To: obm-l@mat.puc-rio.br
 
 2011/2/8 Henrique Rennó henrique.re...@gmail.com:
  Na sequência 1/1, 3/2, 7/5, 17/12, 41/29, ..., o denominador de cada
  termo a partir do segundo é a soma do numerador mais denominador
  anterior e o numerador é a soma de duas vezes o denominador mais
  numerador anterior. Essa sequência converge para raíz_quadrada(2).
  Como isso pode ser demonstrado?
 Oi Henrique. Uma das coisas importantes nesse tipo de problemas é dar
 nome aos burros. Assim, seja x_n a seqüência dos denominadores, y_n a
 dos numeradores. Você quer calcular o limite y_n/x_n. Agora, veja que
 você tem uma recorrência de x_n e y_n em função deles mesmos. Veja que
 dá
 
 x_{n+1} = x_n + y_n
 y_{n+1} = 2*x_n + y_n
 
 Mas o que você quer é o quociente. Vejamos o que dá pra fazer com y_n+1/x_n+1:
 
 y_{n+1}/x_{n+1} = ( 2*x_n + y_n ) / (x_n + y_n ) = 1 + x_n / (x_n +
 y_n ) = 1 + 1/(1 + y_n/x_n)
 
 Agora, *se* existir um limite L para y_n/x_n, será o mesmo que para
 y_{n+1}/x_{n+1}. Isso dá uma equação do segundo grau em L, você
 resolve, e pronto.
 
 Mas isso ainda não prova que converge ! Para provar a convergência, é preciso
 1/ Fazer as contas com épsilons e n = N, e ver que dá certo pra todo
 épsilon, ou
 2/ Tentar mostrar uma propriedade que faça a convergência ficar clara.
 
 Eu deixo você provar usando o método 1 (não é tão difícil assim: uma
 vez que você sabe qual é o limite, é mais uma questão de estimar o
 erro na etapa n, e tentar calcular como ele vai diminuir quando você
 fizer a próxima operação! Dica: estime (y_n/x_n)^2 - 2 a cada vez,
 você vai ver que o erro divide por mais do que 3).
 
 O método 2 é o seguinte: veja que a seqüência é limitada: 0  x_n 
 y_n  2*x_n (veja que y_{n+1} = 2x_n + y_n  2(x_n + y_n) =
 2*x_{n+1}), o que quer dizer que y_n/x_n está sempre entre 1 e 2. Bom,
 a idéia é tentar provar que a seqüência é monótona (a gente não tem
 tanto teorema assim que ajuda a provar a convergência das coisas...).
 Mas nem vale a pena tentar: 1/1=1, 3/2=1,5, 7/5 = 1,4, e o outro 17/12
 é muito chato pra fazer de cabeça. Mas você pode tentar outra coisa...
 ela pode ficar oscilando em volta do limite.
 
 Vamos calcular y_{n+2}/x_{n+2} (nunca perca a coragem, e até agora as
 contas foram poucas!)
 Isso dá (4 x_n + 3 y_n)/(3 x_n + 2 y_n). Fatorando y_n/x_n (afinal, é
 com o quê a gente quer comparar), obtemos  y_{n+2}/x_{n+2} = (y_n/x_n)
 * (3 + 4 x_n/y_n) / (3 + 2 y_n/x_n). O que dá um fator multiplicativo
 (3 + 4/a) / (3 + 2a), onde a = y_n/x_n. Suponha que a  raiz(2). Neste
 caso, a*a  2, logo 2a  4/a, o que faz um fator menor do que 1. Por
 outro lado, se a  raiz(2), o fator é maior do que 1.
 
 O que quer dizer que temos quase tudo para ter uma seqüência que fica
 oscilando entre o limite raiz(2).
 
 Falta só provar que o y_{n+2}/x_{n+2} continua menor do que raiz(2),
 mesmo depois de aumentar, se y_n/x_n for menor do que raiz(2), e
 reciprocamente no outro caso. Isso é fácil de ver assim : suponha que
 y_n/x_n  raiz(2). Vejamos o que acontece com y_{n+1}/x_{n+1} = 1 +
 1/(1 + y_n/x_n)  1 + 1/(1 + raiz(2)) = 1 + (raiz(2) - 1)/(2-1) =
 raiz(2). Oba! E como mudando y_n/x_n  raiz(2) a desigualdade na
 recorrência muda de sinal, a gente provou:
 1/ A seqüência fica alternadamente maior / menor do que raiz(2)
 2/ A cada duas etapas (ou seja, quando o sinal volta a ser o mesmo),
 está mais perto
 3/ Agora, pegue o limite de cada uma das seqüências (a crescente e a
 decrescente, par / ímpar se você prefere), que existe porque é
 limitado e monótono (O teorema), eles satisfazem a mesma equação L =
 L(3 + 4/L)/(3 + 2L), ou seja 3 + 4/L = 3 + 2L, ou seja L^2 = 2, e são
 ambos positivos. Fim!
 
 
 Depois de escrever isso tudo, eu acho que o método 1 foi o mais
 fácil... (ok, precisa da idéia dos quadrados, mas é bastante razoável,
 né?). A vantagem do método 2 é que ele funciona em muitos casos em que
 é difícil obter uma estimativa da convergência!
 
 Abraços,
 -- 
 Bernardo Freitas Paulo da Costa
 
 

[obm-l] RE: [obm-l] Re: [obm-l] Convergência

2011-02-08 Por tôpico Luís Lopes

Sauda,c~oes, 

Obrigado Bernardo por tal solução. Devemos reconhecer 
e apreciar a colaboração (muitas) de membros como você. 

Eu procederia da seguinte maneira: seja o sistema 

x_{n+1} = x_n + y_n
y_{n+1} = 2*x_n + y_n

Resolvendo o sistema acima (alguém sabe como fazer isso?) 
obtemos x_n e y_n e daí a_n =  y_n/x_n. 

Então tendo a_n usamos um daqueles testes manjados para 
testar a convergência de séries (positivas). Será que daria certo? 

[]'s 
Luís 




 Date: Tue, 8 Feb 2011 16:54:04 +0100
 Subject: [obm-l] Re: [obm-l] Convergência
 From: bernardo...@gmail.com
 To: obm-l@mat.puc-rio.br
 
 2011/2/8 Henrique Rennó henrique.re...@gmail.com:
  Na sequência 1/1, 3/2, 7/5, 17/12, 41/29, ..., o denominador de cada
  termo a partir do segundo é a soma do numerador mais denominador
  anterior e o numerador é a soma de duas vezes o denominador mais
  numerador anterior. Essa sequência converge para raíz_quadrada(2).
  Como isso pode ser demonstrado?
 Oi Henrique. Uma das coisas importantes nesse tipo de problemas é dar
 nome aos burros. Assim, seja x_n a seqüência dos denominadores, y_n a
 dos numeradores. Você quer calcular o limite y_n/x_n. Agora, veja que
 você tem uma recorrência de x_n e y_n em função deles mesmos. Veja que
 dá
 
 x_{n+1} = x_n + y_n
 y_{n+1} = 2*x_n + y_n
 
 Mas o que você quer é o quociente. Vejamos o que dá pra fazer com y_n+1/x_n+1:
 
 y_{n+1}/x_{n+1} = ( 2*x_n + y_n ) / (x_n + y_n ) = 1 + x_n / (x_n +
 y_n ) = 1 + 1/(1 + y_n/x_n)
 
 Agora, *se* existir um limite L para y_n/x_n, será o mesmo que para
 y_{n+1}/x_{n+1}. Isso dá uma equação do segundo grau em L, você
 resolve, e pronto.
 
 Mas isso ainda não prova que converge ! Para provar a convergência, é preciso
 1/ Fazer as contas com épsilons e n = N, e ver que dá certo pra todo
 épsilon, ou
 2/ Tentar mostrar uma propriedade que faça a convergência ficar clara.
 
 Eu deixo você provar usando o método 1 (não é tão difícil assim: uma
 vez que você sabe qual é o limite, é mais uma questão de estimar o
 erro na etapa n, e tentar calcular como ele vai diminuir quando você
 fizer a próxima operação! Dica: estime (y_n/x_n)^2 - 2 a cada vez,
 você vai ver que o erro divide por mais do que 3).
 
 O método 2 é o seguinte: veja que a seqüência é limitada: 0  x_n 
 y_n  2*x_n (veja que y_{n+1} = 2x_n + y_n  2(x_n + y_n) =
 2*x_{n+1}), o que quer dizer que y_n/x_n está sempre entre 1 e 2. Bom,
 a idéia é tentar provar que a seqüência é monótona (a gente não tem
 tanto teorema assim que ajuda a provar a convergência das coisas...).
 Mas nem vale a pena tentar: 1/1=1, 3/2=1,5, 7/5 = 1,4, e o outro 17/12
 é muito chato pra fazer de cabeça. Mas você pode tentar outra coisa...
 ela pode ficar oscilando em volta do limite.
 
 Vamos calcular y_{n+2}/x_{n+2} (nunca perca a coragem, e até agora as
 contas foram poucas!)
 Isso dá (4 x_n + 3 y_n)/(3 x_n + 2 y_n). Fatorando y_n/x_n (afinal, é
 com o quê a gente quer comparar), obtemos  y_{n+2}/x_{n+2} = (y_n/x_n)
 * (3 + 4 x_n/y_n) / (3 + 2 y_n/x_n). O que dá um fator multiplicativo
 (3 + 4/a) / (3 + 2a), onde a = y_n/x_n. Suponha que a  raiz(2). Neste
 caso, a*a  2, logo 2a  4/a, o que faz um fator menor do que 1. Por
 outro lado, se a  raiz(2), o fator é maior do que 1.
 
 O que quer dizer que temos quase tudo para ter uma seqüência que fica
 oscilando entre o limite raiz(2).
 
 Falta só provar que o y_{n+2}/x_{n+2} continua menor do que raiz(2),
 mesmo depois de aumentar, se y_n/x_n for menor do que raiz(2), e
 reciprocamente no outro caso. Isso é fácil de ver assim : suponha que
 y_n/x_n  raiz(2). Vejamos o que acontece com y_{n+1}/x_{n+1} = 1 +
 1/(1 + y_n/x_n)  1 + 1/(1 + raiz(2)) = 1 + (raiz(2) - 1)/(2-1) =
 raiz(2). Oba! E como mudando y_n/x_n  raiz(2) a desigualdade na
 recorrência muda de sinal, a gente provou:
 1/ A seqüência fica alternadamente maior / menor do que raiz(2)
 2/ A cada duas etapas (ou seja, quando o sinal volta a ser o mesmo),
 está mais perto
 3/ Agora, pegue o limite de cada uma das seqüências (a crescente e a
 decrescente, par / ímpar se você prefere), que existe porque é
 limitado e monótono (O teorema), eles satisfazem a mesma equação L =
 L(3 + 4/L)/(3 + 2L), ou seja 3 + 4/L = 3 + 2L, ou seja L^2 = 2, e são
 ambos positivos. Fim!
 
 
 Depois de escrever isso tudo, eu acho que o método 1 foi o mais
 fácil... (ok, precisa da idéia dos quadrados, mas é bastante razoável,
 né?). A vantagem do método 2 é que ele funciona em muitos casos em que
 é difícil obter uma estimativa da convergência!
 
 Abraços,
 -- 
 Bernardo Freitas Paulo da Costa
 
 =
 Instruções para entrar na lista, sair da lista e usar a lista em
 http://www.mat.puc-rio.br/~obmlistas/obm-l.html
 =
  

RE: [obm-l] distancia no trapezio

2011-01-11 Por tôpico Luís Lopes

Sauda¸c~oes, oi Carlos Victor, 

Fiz como vc, soh que tracei por C a paralela ao lado AD. Mas n~ao observei 
que o triângulo CBE é isósceles com base BC. 

Obrigado. 

Abraços, 

Luis 


Date: Sun, 9 Jan 2011 18:07:16 -0200
Subject: Re: [obm-l] distancia no trapezio
From: victorcar...@globo.com
To: obm-l@mat.puc-rio.br

Olá  Luis,

Trace de D uma paralela ao lado BC que encontra AB por exemplo em E. Observe 
que BE = 4 e o triângulo EDA é isósceles  e, donde EA = 6. Logo AB = 10 , ok ? 
. Verifique se esta ideia está correta .


Abraços 

Carlos Victor

Em 9 de janeiro de 2011 13:56, Luís Lopes qed_te...@hotmail.com escreveu:






Sauda¸c~oes, 

Pediram-me para resolver o seguinte problema. 

No trapezio ABCD, CD=4 e DA=6. E tambem B=\alpha e D=2\alpha. 
Calcular AB. 

O desenho do trapezio eh 

  C o---o D 




B o---o A 


Abra¸cos, 
Luis 

  

  

[obm-l] RE: [obm-l] indução finita

2011-01-09 Por tôpico Luís Lopes

Sauda¸c~oes, oi Eder, 

Embora não usando a sugestão do Elon, nos  exercícios 11 e 56 do 
Manual de Indução (ver www.escolademestres.com) demonstro 
tal resultado. 

E acredito que no  exercício 12 você encontre elementos para fazer a 
demonstração como sugerido. 

Abraços, 
Luis 



Date: Sun, 9 Jan 2011 05:56:07 -0800
From: eder_it...@yahoo.com.br
Subject: [obm-l] indução finita
To: obm-l@mat.puc-rio.br

Pessoal,
Depois de passar muito tempo meditando sobre o exercício abaixo (consta num 
artigo do Elon Lages Lima publicado na Eureka), resolvi enviar para a lista. Se 
alguém puder resolver, fico muito agradecido... Eis a questão:
Para todo n em N, ponha x_n = { (n+1)^2 / [n(n+2)] }^n e prove por indução que 
se tem x_n  (n+2)/(n+1). Conclua que a seqüência de termo geral x_n 
=[(n+1)/n]^n é crescente.
Sugestão: x_(n+1)=[(n+2)/(n+1)]^3.[n/(n+3)].x_n.   (será que está certo 
isso???).
Obrigado,
Eder
  

[obm-l] distancia no trapezio

2011-01-09 Por tôpico Luís Lopes

Sauda¸c~oes, 

Pediram-me para resolver o seguinte problema. 

No trapezio ABCD, CD=4 e DA=6. E tambem B=\alpha e D=2\alpha. 
Calcular AB. 

O desenho do trapezio eh 

  C o---o D 



B o---o A 


Abra¸cos, 
Luis 

  

[obm-l] RE: [obm-l] Re: [obm -l] RE: [obm-l] Re: [obm-l] RE: [obm-l] Re: [obm-l] RE: [obm -l] Limite de série

2010-11-18 Por tôpico Luís Lopes

Sauda,c~oes, oi Lucas, 

Gostaria de voltar ao assunto. 

Não me importarei se não entender a solução. Mas realmente 
gostaria de vê-la. Ou se não for possível (será mesmo que podemos 
calcular a soma da série??) gostaria de ter pelo menos a resposta. 

Se vc preferir, favor pedir pro seu professor me escrever diretamente. 

[]'s 
Luís 


From: luca...@dcc.ufba.br
Date: Thu, 18 Nov 2010 06:34:24 -0300
Subject: [obm-l] Re: [obm-l] RE: [obm-l] Re: [obm-l] RE: [obm-l] Re: [obm-l] 
RE: [obm-l] Limite de série
To: obm-l@mat.puc-rio.br

2010/11/16 Luís Lopes qed_te...@hotmail.com







Sauda,c~oes, oi Lucas, 

Entendido. Aguardo os comentários do seu professor. 
Eu falei com ele e parece que encontrar a soma da série pode envolver 
conhecimentos de análise funcional (se não me engano) que estão acima da alçada 
de um estudante de cálculo C. Então (acho) não poderei dar mais detalhes sobre 
a solução, infelizmente. =/


(isso sugere que essa série não devia estar na lista de exercícios...)

-- 
[]'s
Lucas
  

[obm-l] RE: [obm-l] Re: [obm -l] RE: [obm-l] Re: [obm-l] RE: [obm-l] Limite de série

2010-11-16 Por tôpico Luís Lopes

Sauda,c~oes, oi Lucas, 

Entendido. Aguardo os comentários do seu professor. 

[]'s 
Luís 


From: luca...@dcc.ufba.br
Date: Mon, 15 Nov 2010 21:19:38 -0300
Subject: [obm-l] Re: [obm-l] RE: [obm-l] Re: [obm-l] RE: [obm-l] Limite de série
To: obm-l@mat.puc-rio.br

2010/11/15 Luís Lopes qed_te...@hotmail.com







Sauda,c~oes, oi Lucas, 

Troquei emails com o prof Rousseau e achar o valor da 
série dada pelo somando arctan(n)/(1+n²) está se revelando 
muito difícil. Inclusive a resposta sen 1 parece errada. 

Vc poderia nos dar alguma dica? Falar com o professor que passou 


o problema, confirmar o valor da série etc? O que foi feito em termos 
de correção e avaliação da lista? 

Eu acredito que é um erro de digitação mesmo.
A lista não é rigidamente corrigida porque serve somente de suporte para a 
disciplina e não como avaliação.



Como há muitos professores da disciplina, deve ser muito difícil encontrar o 
autor desta questão para nos esclarecer, mas vou ver com o meu professor.

Muito obrigado por olhar a questão.

-- 
[]'s


Lucas
  

[obm-l] geometria com 20 graus

2010-11-16 Por tôpico Luís Lopes

Sauda,c~oes, 

Pediram-me a solução do problema abaixo. Como muito provavelmente 
tal problema já apareceu por aqui, pergunto se alguém teria a solução 
dele à mão. 

Obrigado. 

[]'s 
Luís 




 Prezado Luís mais uma vez venho pedir a sua ajuda na solução do exercicio 
abaixo.
Dado o triângulo ABC ,isósceles, com BÂC medindo 20 graus ,onde AC = AB. 
Pelo vértice B traça-se até AC, BD tal que AD = BC. Calcule o valor do ângulo 
B^DC. 


  

[obm-l] RE: [obm-l] Re: [obm -l] RE: [obm-l] Limi te de série

2010-11-15 Por tôpico Luís Lopes

Sauda,c~oes, oi Lucas, 

Troquei emails com o prof Rousseau e achar o valor da 
série dada pelo somando arctan(n)/(1+n²) está se revelando 
muito difícil. Inclusive a resposta sen 1 parece errada. 

Vc poderia nos dar alguma dica? Falar com o professor que passou 
o problema, confirmar o valor da série etc? O que foi feito em termos 
de correção e avaliação da lista? 

[]'s 
Luís 




From: luca...@dcc.ufba.br
Date: Mon, 8 Nov 2010 09:56:03 -0300
Subject: [obm-l] Re: [obm-l] RE: [obm-l] Limite de série
To: obm-l@mat.puc-rio.br

2010/11/8 Luís Lopes qed_te...@hotmail.com







Sauda,c~oes, 
Oi Lucas, 

Você tem a fonte deste problema? 

E favor confirmar se é mesmo arctan(n)/(1+n²). Poderia ser 
arctan [n/(1+n^2)] ? É uma lista da disciplina de cálculo C da UFBA.


Pode ser baixada aqui: http://www.graphics.ufba.br/unid3lista2010.1.pdf
É a questão 4.m

A propósito, se vc puder responder para arctan[n/(1+n²)] acho que seria útil 
também :-)



-- 
[]'s
Lucas
  

[obm-l] RE: [obm-l] Limite d e série

2010-11-08 Por tôpico Luís Lopes

Sauda,c~oes, 
Oi Lucas, 

Você tem a fonte deste problema? 

E favor confirmar se é mesmo arctan(n)/(1+n²). Poderia ser 
arctan [n/(1+n^2)] ? 

Luís 


From: luca...@dcc.ufba.br
Date: Wed, 3 Nov 2010 21:17:08 -0300
Subject: [obm-l] Limite de série
To: obm-l@mat.puc-rio.br

Olá,

como encontrar o limite da série cuja sequência é arctan(n)/(1+n²)?
-- 
[]'s
Lucas
  

RE: [obm-l] Geometria Olimpica

2010-07-16 Por tôpico Luís Lopes

Sauda,c~oes, 

 

Seria legal conhecer outras soluções dos membros 

da lista e da própria OBM. 

 

Seguem outra solução de ND e correções de APH. 

 

[]'s 

Luis 


=

Lemma 1:

In every triangle:

 

Lemma 1:

GI^2 = (bc+ca+ab)/3 - (a^2+b^2+c^2)/9 - 4Rr

Lemma 2:

If 2b = a + c [: b = a+d, c = a+2d ] == ac = 6Rr

L1 /\ L2 == 9GI^2 = b^2 - ac = (a+d)^2 - a(a+2d) = d^2

== GI = d/3

APH





==

Dear Tuan, 
very good!
Another proof with vectors: 
We have 
GA + GB + GC = 0
a.IA + b.IB + c.IC = 0 or
(a + b + c).IG + a.GA + b.GB + c.GC = 0 or
3b.IG + (a - b)GA + (c - b)GC = 0 or
3b.IG = d.(GA - GC) = d.CA
and hence |IG| = |CA|.d/3b = d/3.
Best regards
Nikos Dergiades


 


From: qed_te...@hotmail.com
To: obm-l@mat.puc-rio.br
Subject: RE: [obm-l] Geometria Olimpica
Date: Thu, 15 Jul 2010 20:06:14 +



Sauda,c~oes, 

Três soluções de um outro grupo. 

[]'s 
Luis 

== 1)

 Dear Luis,
 Let b = a + d, c = b + d, or a + c = 2b
 then s = (a + b + c) / 2 = 3b/2, s - b = b/2 and since
 s.r = (s - b) rb we get that rb = 3r
 where r, rb are the radi of incircle and B_excircle.
 
 The line IG has equation in barycentrics
 (b - c)x + (c - a)y + (a - b)z = 0 or
 -x + 2y - z = 0 and meets the line BC at D (0 : 1 : 2)
 which means that BD/DC = 2 and the line IG is 
 parallel to AC. If M is the midpoint of AC and the 
 line BI meets AC at J then 
 CJ = ab/(a + c) = a/2
 MJ = b/2 - a/2 = d/2 and
 GI = 2MJ/3 = d/3.
 
 The Nagel point Na is known that lies on line IG
 and if BNa meets AC at K then 
 AK = s - c = 3b/2 - c and 
 KM = AM - AK = b/2 - (3b/2 - c) = c - b = d
 Hence NaG = 2KM/3 = 2d/3
 
 Best regards
 Nikos Dergiades





== 2)

Dear Luis 

Lemma 1:

In every triangle:

GI^2 = (bc+ca+ab)/3 - (a^2+b^2+c2)/9 - 4Rr

Lemma 2:

If 2b = a + c [: a = b+d, c = b+2d ] == ac = 6Rr

L1 /\ L2 == 9GI^2 = b^2 - 3ac = (c-a)^2 ==

3GI = c-a = d == GI = d/3

APH
 

== 3)

Dear Luis and Nikos,

We can solve the problem by construction triangle ABC (b = a+d = c-d) as 
following:

Choose b = AC as one segment.
X as any point on segment AC and CX = d.
X1 = reflection of X in C
X2 = reflection of X in midpoint of AC
B = intersection of two circles centered at C passing X2 and centered at A 
passing X1
Y1 = midpoint of BX1
Y2 = midpoint of BX2
A1 = midpoint of BC
C1 = midpoint of AB
I = intesection of AY1 and CY2
G = intesection of AA1 and CC1

From this construction: GI//AC and 
GI = 2/3*A1Y1 = 2/3*1/2*CX1 = 1/3*CX1 = d/3

Best regards,
Bui Quang Tuan



From: marconeborge...@hotmail.com
To: obm-l@mat.puc-rio.br
Subject: RE: [obm-l] Geometria Olimpica
Date: Tue, 13 Jul 2010 21:51:15 +



Muito ´´bonito´´ mesmo.Seria muito interessante ver soluções diferentes 
postadas aqui neste fabuloso espaço.
 


Date: Tue, 13 Jul 2010 14:36:06 -0700
From: luizfelipec...@yahoo.com.br
Subject: RE: [obm-l] Geometria Olimpica
To: obm-l@mat.puc-rio.br






Estou enviando, pois achei o problema muito bonito.
 
Abs
Felipe

--- Em ter, 13/7/10, Thiago Tarraf Varella thiago_...@hotmail.com escreveu:


De: Thiago Tarraf Varella thiago_...@hotmail.com
Assunto: RE: [obm-l] Geometria Olimpica
Para: OBM Lista obm-l@mat.puc-rio.br
Data: Terça-feira, 13 de Julho de 2010, 13:57




Você está apenas comentando com agente pois achou ele legal ou você quer ajuda 
na resolução? 
Thiago



Date: Mon, 12 Jul 2010 10:48:57 -0700
From: luizfelipec...@yahoo.com.br
Subject: [obm-l] Geometria Olimpica
To: obm-l@mat.puc-rio.br







Pessoal,
 
Segue problema da OBM :
 
Os lados de um triângulo qualquer estão em uma P.A de razão r. Calcular a 
distância do incentro ao baricentro deste triangulo, em função de r.
 
Abs
Felipe



O INTERNET EXPLORER 8 DÁ DICAS DE SEGURANÇA PARA VOCÊ SAIBA MAIS! 



FIQUE MAIS PROTEGIDO ENQUANTO FAZ DOWNLOADS INSTALE GRÁTIS O INTERNET EXPLORER 
8. 


LEVE SEU MESSENGER PARA ONDE VOCÊ ESTIVER PELO SEU CELULAR. CLIQUE E VEJA COMO 
FAZER. 
_
CONVERSE COM SEUS AMIGOS E OS VEJA PELA WEBCAM NO MESSENGER. CLIQUE AQUI E VEJA 
COMO.
http://www.windowslive.com.br/public/tip.aspx/view/84?product=2ocid=WLCRM:Live:Hotmail:Tagline:senDimensao:CONVERSECO85:-

RE: [obm-l] Geometria Olimpica

2010-07-15 Por tôpico Luís Lopes

Sauda,c~oes, 

Três soluções de um outro grupo. 

[]'s 
Luis 

== 1)


 Dear Luis,
 Let b = a + d, c = b + d, or a + c = 2b
 then s = (a + b + c) / 2 = 3b/2, s - b = b/2 and since
 s.r = (s - b) rb we get that rb = 3r
 where r, rb are the radi of incircle and B_excircle.
 
 The line IG has equation in barycentrics
 (b - c)x + (c - a)y + (a - b)z = 0 or
 -x + 2y - z = 0 and meets the line BC at D (0 : 1 : 2)
 which means that BD/DC = 2 and the line IG is 
 parallel to AC. If M is the midpoint of AC and the 
 line BI meets AC at J then 
 CJ = ab/(a + c) = a/2
 MJ = b/2 - a/2 = d/2 and
 GI = 2MJ/3 = d/3.
 
 The Nagel point Na is known that lies on line IG
 and if BNa meets AC at K then 
 AK = s - c = 3b/2 - c and 
 KM = AM - AK = b/2 - (3b/2 - c) = c - b = d
 Hence NaG = 2KM/3 = 2d/3
 
 Best regards
 Nikos Dergiades





== 2)

Dear Luis 

Lemma 1:

In every triangle:

GI^2 = (bc+ca+ab)/3 - (a^2+b^2+c2)/9 - 4Rr

Lemma 2:

If 2b = a + c [: a = b+d, c = b+2d ] == ac = 6Rr

L1 /\ L2 == 9GI^2 = b^2 - 3ac = (c-a)^2 ==

3GI = c-a = d == GI = d/3

APH
 

== 3)

Dear Luis and Nikos,

We can solve the problem by construction triangle ABC (b = a+d = c-d) as 
following:

Choose b = AC as one segment.
X as any point on segment AC and CX = d.
X1 = reflection of X in C
X2 = reflection of X in midpoint of AC
B = intersection of two circles centered at C passing X2 and centered at A 
passing X1
Y1 = midpoint of BX1
Y2 = midpoint of BX2
A1 = midpoint of BC
C1 = midpoint of AB
I = intesection of AY1 and CY2
G = intesection of AA1 and CC1

From this construction: GI//AC and 
GI = 2/3*A1Y1 = 2/3*1/2*CX1 = 1/3*CX1 = d/3

Best regards,
Bui Quang Tuan



From: marconeborge...@hotmail.com
To: obm-l@mat.puc-rio.br
Subject: RE: [obm-l] Geometria Olimpica
Date: Tue, 13 Jul 2010 21:51:15 +



Muito ´´bonito´´ mesmo.Seria muito interessante ver soluções diferentes 
postadas aqui neste fabuloso espaço.
 


Date: Tue, 13 Jul 2010 14:36:06 -0700
From: luizfelipec...@yahoo.com.br
Subject: RE: [obm-l] Geometria Olimpica
To: obm-l@mat.puc-rio.br






Estou enviando, pois achei o problema muito bonito.
 
Abs
Felipe

--- Em ter, 13/7/10, Thiago Tarraf Varella thiago_...@hotmail.com escreveu:


De: Thiago Tarraf Varella thiago_...@hotmail.com
Assunto: RE: [obm-l] Geometria Olimpica
Para: OBM Lista obm-l@mat.puc-rio.br
Data: Terça-feira, 13 de Julho de 2010, 13:57




Você está apenas comentando com agente pois achou ele legal ou você quer ajuda 
na resolução? 
Thiago



Date: Mon, 12 Jul 2010 10:48:57 -0700
From: luizfelipec...@yahoo.com.br
Subject: [obm-l] Geometria Olimpica
To: obm-l@mat.puc-rio.br







Pessoal,
 
Segue problema da OBM :
 
Os lados de um triângulo qualquer estão em uma P.A de razão r. Calcular a 
distância do incentro ao baricentro deste triangulo, em função de r.
 
Abs
Felipe



O INTERNET EXPLORER 8 DÁ DICAS DE SEGURANÇA PARA VOCÊ SAIBA MAIS! 



FIQUE MAIS PROTEGIDO ENQUANTO FAZ DOWNLOADS INSTALE GRÁTIS O INTERNET EXPLORER 
8. 
_
TRANSFORME SUAS FOTOS EM EMOTICONS PARA O MESSENGER. CLIQUE AQUI PARA COMEÇAR.
http://ilm.windowslive.com.br/?ocid=ILM:Live:Hotmail:Tagline:senDimensao:TRANSFORME78:-

RE: [obm-l] Geometria Olimpica

2010-07-15 Por tôpico Luís Lopes

Sauda,c~oes, 

 

Três soluções de um outro grupo. 

 

[]'s 

Luis 

 

== 1)


 Dear Luis,
 Let b = a + d, c = b + d, or a + c = 2b
 then s = (a + b + c) / 2 = 3b/2, s - b = b/2 and since
 s.r = (s - b) rb we get that rb = 3r
 where r, rb are the radi of incircle and B_excircle.
 
 The line IG has equation in barycentrics
 (b - c)x + (c - a)y + (a - b)z = 0 or
 -x + 2y - z = 0 and meets the line BC at D (0 : 1 : 2)
 which means that BD/DC = 2 and the line IG is 
 parallel to AC. If M is the midpoint of AC and the 
 line BI meets AC at J then 
 CJ = ab/(a + c) = a/2
 MJ = b/2 - a/2 = d/2 and
 GI = 2MJ/3 = d/3.
 
 The Nagel point Na is known that lies on line IG
 and if BNa meets AC at K then 
 AK = s - c = 3b/2 - c and 
 KM = AM - AK = b/2 - (3b/2 - c) = c - b = d
 Hence NaG = 2KM/3 = 2d/3
 
 Best regards
 Nikos Dergiades
  



== 2)

Dear Luis 

Lemma 1:

In every triangle:

GI^2 = (bc+ca+ab)/3 - (a^2+b^2+c2)/9 - 4Rr

Lemma 2:

If 2b = a + c [: a = b+d, c = b+2d ] == ac = 6Rr

L1 /\ L2 == 9GI^2 = b^2 - 3ac = (c-a)^2 ==

3GI = c-a = d == GI = d/3

APH
 

== 3)

Dear Luis and Nikos,

We can solve the problem by construction triangle ABC (b = a+d = c-d) as 
following:

Choose b = AC as one segment.
X as any point on segment AC and CX = d.
X1 = reflection of X in C
X2 = reflection of X in midpoint of AC
B = intersection of two circles centered at C passing X2 and centered at A 
passing X1
Y1 = midpoint of BX1
Y2 = midpoint of BX2
A1 = midpoint of BC
C1 = midpoint of AB
I = intesection of AY1 and CY2
G = intesection of AA1 and CC1

From this construction: GI//AC and 
GI = 2/3*A1Y1 = 2/3*1/2*CX1 = 1/3*CX1 = d/3

Best regards,
Bui Quang Tuan


 

From: marconeborge...@hotmail.com
To: obm-l@mat.puc-rio.br
Subject: RE: [obm-l] Geometria Olimpica
Date: Tue, 13 Jul 2010 21:51:15 +



Muito ´´bonito´´ mesmo.Seria muito interessante ver soluções diferentes 
postadas aqui neste fabuloso espaço.
 


Date: Tue, 13 Jul 2010 14:36:06 -0700
From: luizfelipec...@yahoo.com.br
Subject: RE: [obm-l] Geometria Olimpica
To: obm-l@mat.puc-rio.br






Estou enviando, pois achei o problema muito bonito.
 
Abs
Felipe

--- Em ter, 13/7/10, Thiago Tarraf Varella thiago_...@hotmail.com escreveu:


De: Thiago Tarraf Varella thiago_...@hotmail.com
Assunto: RE: [obm-l] Geometria Olimpica
Para: OBM Lista obm-l@mat.puc-rio.br
Data: Terça-feira, 13 de Julho de 2010, 13:57




Você está apenas comentando com agente pois achou ele legal ou você quer ajuda 
na resolução? 
Thiago



Date: Mon, 12 Jul 2010 10:48:57 -0700
From: luizfelipec...@yahoo.com.br
Subject: [obm-l] Geometria Olimpica
To: obm-l@mat.puc-rio.br







Pessoal,
 
Segue problema da OBM :
 
Os lados de um triângulo qualquer estão em uma P.A de razão r. Calcular a 
distância do incentro ao baricentro deste triangulo, em função de r.
 
Abs
Felipe



O INTERNET EXPLORER 8 DÁ DICAS DE SEGURANÇA PARA VOCÊ SAIBA MAIS! 



FIQUE MAIS PROTEGIDO ENQUANTO FAZ DOWNLOADS INSTALE GRÁTIS O INTERNET EXPLORER 
8. 
_
CONVERSE COM SEUS AMIGOS E OS VEJA PELA WEBCAM NO MESSENGER. CLIQUE AQUI E VEJA 
COMO.
http://www.windowslive.com.br/public/tip.aspx/view/84?product=2ocid=WLCRM:Live:Hotmail:Tagline:senDimensao:CONVERSECO85:-

RE: [obm-l] Divisores

2010-06-28 Por tôpico Luís Lopes

Sauda,c~oes, 

 

Na última RPM vemos o seguinte problema: 

 

Dado n um número inteiro positivo, seja f(n) a média aritmética 

dos (todos) divisores positivos de n.

 

a) mostre que \sqrt{n} \leq f(n) \leq (n+1)/2. 

 

b) encontre todos os inteiros positivos tais que f(n) = 91/9. 

 

[]'s 

Luís 





Date: Sat, 26 Jun 2010 00:43:42 -0300
Subject: Re: [obm-l] Divisores
From: ralp...@gmail.com
To: obm-l@mat.puc-rio.br


Um dos divisores eh n, outro eh 1 -- por enquanto, a media eh (n+1)/2.
 
Agora, se houver outros, eles sao todos menores ou iguais a n/2  (n+1)/2, 
diminuindo mais esta media.
 
(Nao eh dificil escrever este argumento de uma maneira mais formal e algebrica, 
mas a ideia eh essa ai).
 
Abraco, Ralph.


2010/6/25 marcone augusto araújo borges marconeborge...@hotmail.com


Seja n um número inteiro positivo e M a média aritmética dos divisores 
positivos de n.Como demonstra que M  = (n+1)/2? 


TRANSFORME-SE EM PERSONAGENS ENGRAÇADOS COM O SITE DE I LOVE MESSENGER. VEJA 
COMO.
  
_
O INTERNET EXPLORER 8 TE AJUDA A FICAR PROTEGIDO DE FRAUDES NA WEB. LEIA MAIS 
AQUI.
http://www.microsoft.com/brasil/windows/internet-explorer/features/dicas.aspx?tabid=1catid=1WT.mc_id=1589

RE: [obm-l] IMO Polinomio irredutivel

2010-06-25 Por tôpico Luís Lopes

Sauda,c~oes, 

 

b) a_0=5. Irredutível (por Eisenstein com p=5)
 
c) a_0=6. Redutível. ??

 

x^2 + 5x + 6 = (x+2)(x+3)

x^3 + 5x^2 + 6 = ?? (*)

Mostre que se m é composto a fatoração de polinômios 
mód. m não é única. (**) 

 

Como provar (**) e depois usar em (*) ? 

 

[]'s 

Luís 


 


 



From: qed_te...@hotmail.com
To: obm-l@mat.puc-rio.br
Subject: [obm-l] IMO Polinomio irredutivel
Date: Thu, 24 Jun 2010 22:20:17 +



Sauda,c~oes, oi Johann Dirichlet, 
 
Fiz reply e a mensagem não foi. Mando como nova msg. 

Vc(s) saberia dizer o ano da IMO deste problema? 
Haveria uma outra solução para este problema? 
 
O mesmo problema x^n + 5x^{n-1} + a_0 para 
o termo independente a_0 igual a 4, 5 e 6. 
 
a) a_0=4. Redutível para n par pois 1 - 5 + 4 = 0
 
b) a_0=5. 
 
c) a_0=6. 
 
Juntamente com o problema 
 
Mostre que se m é composto a fatoração de polinômios 
mód. m não é única. 
 
[]s 
Luís 




VEJA TODOS OS SEUS EMAILS DE VÁRIAS CONTAS COM UM SÓ LOGIN. CLIQUE AQUI E VEJA 
COMO.
  
_
VEJA SEUS EMAILS ONDE QUER QUE VOCÊ ESTEJA, ACESSE O HOTMAIL PELO SEU CELULAR 
AGORA.
http://celular.windowslive.com.br/hotmail.asp?produto=Hotmailutm_source=Live_Hotmailutm_medium=Taglineutm_content=VEJASEUSEM84utm_campaign=MobileServices

[obm-l] IMO Polinomio irredutivel

2010-06-24 Por tôpico Luís Lopes

Sauda,c~oes, 

 

Na página 27 do livro 21 Aulas de Mat. Olímp. do C. Y. Shine 

encontro o seguinte problema: 

 

Prove que o polinômio x^n + 5x^{n-1} + 3 é irredutível em Q(Z). 

 

Gostaria de ver a solução baseada com o que foi mostrado no 

livro e as referências (fonte e solução) da página do kalva. 

 

Aliás o link para a homepage do kalva do site da Olimp. Paulista de Mat. 

estava quebrado quando tentei entrar. 

 

Obrigado. 

 

[]'s 

Luís 

 
  
_
VEJA SEUS EMAILS ONDE QUER QUE VOCÊ ESTEJA, ACESSE O HOTMAIL PELO SEU CELULAR 
AGORA.
http://celular.windowslive.com.br/hotmail.asp?produto=Hotmailutm_source=Live_Hotmailutm_medium=Taglineutm_content=VEJASEUSEM84utm_campaign=MobileServices

  1   2   3   >